Oletetusti väärin

17.2.2023 klo 18.56, kirjoittaja
Kategoriat: Kosmokseen kirjoitettua , Kosmologia

Kollegani ja ystäväni Will Kinney Buffalon yliopistosta mainitsi hiljattain seikan joka on fyysikoille (ainakin teoreettisille hiukkasfyysikoille ja kosmologeille) selvä, mutta saattaa tulla muille yllätyksenä: tutustuessaan kollegoidensa työhön fyysikot lähtevät usein siitä oletuksesta, että se on väärin.

Tähän kyseenalaistamisen kulttuuriin kuuluu toisaalta sekin, että voi nopeasti muuttaa kantaansa ilman että sitä pidetään nolona. Koska tutkimuksen matemaattinen paikkansapitävyys ei riipu kenenkään mielipiteistä, tämä on myös oppimisen menetelmä, jolla voi vakuuttaa itsensä siitä, että asia jota pitää vääränä onkin oikein – tai löytää sen heikot kohdat.

Suhtautumistapaan liittyy se, että vaikka teoreetikot voivat arvostella toisten ideoita voimakkaasti niihin törmätessään, he eivät tyypillisesti katso tarpeelliseksi ruveta erikseen kumoamaan virheellisinä pitämiään tutkimuksia. Yleensä niihin viitataan lähinnä kintaalla.

Koetulosten kohdalla suhtautuminen on toinen: jos joku raportoi havainnosta, joka osoittaa uuden teorian oikeaksi, väitteen oikeellisuutta ruoditaan tarkkaan. Takana on sama syy kuin ylimalkaisuudessa teoreettisten väitteiden suhteen: havainnot lopulta osoittavat, mitkä teoreettiset ideat ovat oikein ja mitkä eivät. Havaintotulosten suhteen pitää siis olla huolellinen, mutta teoreettisten ideoiden heittelemisestä ei ole suurta haittaa.

Teoreettisilla fyysikoilla on laaja vapaus valita aiheensa, vaikka rahoitusjärjestelmä rajoittaakin tutkimusta yhä enemmän. Hajanainen eri suuntiin vaeltaminen on historiallisesti tuottanut merkittäviä läpimurtoja, vaikka suurin osa reiteistä päätyykin umpikujaan. Kuten filosofi Ludwig Wittgenstein on todennut: jos ihmiset eivät ikinä tekisi typeriä asioita, mitään älykästä ei koskaan tulisi tehtyä.

Fysiikan historiattomuus pelastaa paljon. Oikeat teoriat sisältävät edeltäjänsä, eikä vääriin tarvitse koskaan palata. Tämän onnellisen seikan haittapuoli on se että, antropologi Sharon Traweekin sanoin, fyysikoiden käsitys alansa historiasta on lyhyt hagiografia ja lista ihmeitä. Tilanne on erilainen kuin ihmistieteissä, missä uusi tutkimus ei syrjäytä aiempaa, vaan menneisyys kulkee uuden tiedon rinnalla.

Ymmärryksen puute siitä, että teoreettisilla fyysikoilla on paljon ideoita, joista vain pieni osa osoittautuu oikeaksi, voi merkittävästi vääristää tiedeuutisten välittämää kuvaa tieteestä. Ongelmaa korostaa se, että suurin osa fyysikoista ei koe tarpeelliseksi julkisesti arvostella virheitä, jotka aika kuitenkin pian hautaa.

Vaikuttaa myös siltä, että jotkut toimittajat luulevat, että kaikki vertaisarvioidut tieteelliset artikkelit pitävät paikkansa. Vertaisarviointi karsii kuitenkin korkeintaan ilmeiset virheet – eikä aina edes niitä. Joka päivä ilmestyy kymmeniä kosmologian artikkeleita, ja niiden laatu on hyvin vaihteleva.

Toissapäivänä muun muassa New Scientist ja The Guardian uutisoivat tutkimuksesta, jonka mukaan mustat aukot selittävät pimeän energian, eli sen miksi maailmankaikkeuden laajeneminen kiihtyy. Lukemistani jutuista vähiten huono oli Guardianissa, missä oli mukana kommentit tutkimusryhmän ulkopuoliselta fyysikolta, joka heitti kylmää vettä poskettomille väitteille. Tämä ei kuitenkaan pelasta sitä, että ei ollut mitään syytä tehdä uutista alun perinkään, koska tutkimuksen väitteet ovat villiä spekulaatiota ilman kunnollista matemaattista tai fysikaalista pohjaa. Tekniikan Maailman mukaan on kuitenkin peräti saatu ”todisteita” siitä, että ”pimeää energiaa syntyy mustissa aukoissa”.

Olen monesti (täällä, täällä, täällä, täällä, täällä, täällä, täällä ja täällä) kirjoittanut tiedeuutisoinnin ongelmista. Niitä on varmaan kaikissa suomalaisissa tiedotusvälineissä, joka kirjoittavat tieteestä yleistajuisesti – vähiten julkaisuissa, jotka ovat tiedeyhteisöä lähellä, kuten Yliopisto-lehdessä. Tällä kertaa on kuitenkin syytä kehua joskus arvostelemiani Helsingin Sanomia ja Yleä siitä, miten ne ovat uutisoineet tästä tutkimuksesta, eli eivät mitenkään.

19 kommenttia “Oletetusti väärin”

  1. Tämä on mielenkiintoinen aihe. Tuon tähän yhden datapisteen. Kun aloin opiskella fysiikkaa, ajattelin jostain syystä että väärän ja oikean suhde on keskimäärin 50/50. Olin kuitenkin huomaavinani että oli – ja on edelleen – joitain kollegoita, joiden mielestä ihmisen ”kuuluu” luottaa referoituihin papereihin.

  2. Cargo sanoo:

    ”Hajanainen eri suuntiin vaeltaminen on historiallisesti tuottanut merkittäviä läpimurtoja, vaikka suurin osa reiteistä päätyykin umpikujaan.”

    Tällaista sosiologista asiantilaa kuvailee kaiketi parhaiten Friedrich Nietzsche: ”Synnyttääksesi tanssivan tähden, sisälläsi tulee olla kaaosta.”

    Mutta joo, perustuuko tuo artikkeli todellisiin havaintoihin mustien aukkojen koosta, ja voiko tuon spekuloidun pimeän energian taustalla olla jokin kvanttigravitaatioefekti? (… jossa esim. vetävän massan kaarevuus aiheuttaa mahdollisen laajalle levittyvän vastakkaisen kaarevuuden. Jos hyvin kaarevalla aallolla on taipumus levitä, niin jotain vastaavaa voisi ilmetä myös avaruuden sisällä.)

    1. Syksy Räsänen sanoo:

      Tutkijat julkaisivat kaksi artikkelia.

      Ensimmäisessä he vertailivat mustien aukkojen massoja ei aikoina, ja totesivat, että myöhäisemmissä galakseissa näkyvät aukot ovat raskaampia kuin varhaisissa näkyvät.

      Toisessa he olettivat, että kyse on siitä, että aukkojen massa kasvaa jostain tuntemattomasta syystä, ja osoittivat että tällöin aukkojen massan pitäisi kasvaa verrannollisesti avaruuden tilavuuteen. Tällöin niiden energiatiheys (eli energia/tilavuus) säilyisi vakiona, kuten tyhjön energialla.

      He spekuloivat, että massan kasvu voisi liittyä siihen, että ei tiedetä, miten mustia aukkoja kuvataan laajenevassa avaruudessa. Itse asiassa tämä kuitenkin tiedetään, eikä siihen liity mitään massan kasvua.

      Mielestäni ei ole mielekästä ruveta spekuloimaan, miten väärä idea olla jostain syystä olla oikein.

      1. Eusa sanoo:

        https://bigthink.com/starts-with-a-bang/black-holes-dark-energy/

        Ethan Siegelin paneutuminen aiheeseen on mielekästä luettavaa.

        Ainoat villat, jotka tutkimuksesta löydän, ovat:
        – havaintoihin perustuvuus
        – korrelaatiolöytö alle 5 sigman luotettavuudella

        Näin ollen mitään kausaation päätelmää ei voi tehdä mustien aukkojen merkityksestä; ne voivat kasvaa luonnollisesti ja kausaatio korrelaatiolle (jos se varmistuu) voi löytyä konventionaalisista rakennemekanismeista.

        Huonon tutkimuksen uutisoinnista tekee valitettavan yleinen menettely, jossa hehkutetaan teoreettista erikoisuutta, katsotaan sen vahvistamiselle havainto-olettama ja kuvitellaan osutun oikeaan havaintodatan osuessa olettamaan. Tieteellisesti kestävää on falsifiointi, jossa selitysmallia koetellaan havaintodatalla ja selvitetään pätevyysalue, jonka ulkopuolella se ei ainakaan toimi. Lopulta paradigma on se selitysmalli, jolla on siihen saakka tutkitun perusteella laajin pätevyysalue ja tasavertaisista Occamin partaveitsellä ajeltu paras kauneus eli yksinkertaisuus.

  3. Lentotaidoton sanoo:

    TÄHDET JA AVARUUS 19.2.2023:
    ”Tutkijaryhmä väittää – mustat aukot ovat vastuussa pimeästä energiasta”

    Syksy Räsänen 18.2.2023:

    ”Mielestäni ei ole mielekästä ruveta spekuloimaan, miten väärä idea olla jostain syystä olla oikein”
    .
    Tällaista tämä on. Molemmat Ursan sivuilla. Toki Tähdet ja Avaruus antaa myös linkin kritiikkiin: https://bigthink.com/starts-with-a-bang/black-holes-dark-energy/
    Kehotan jokaista lukaisemaan nimenomaan tämän kritiikkilinkin.

    1. Syksy Räsänen sanoo:

      Kas vain, tosiaan.

      Jutussa lukee: ”Joka tapauksessa nyt julkaistu tutkimus on poikkeuksellisen kiinnostava ja se on herättänyt paljon keskustelua tutkijoiden piirissä.”

      Itse asiassa tällainen täysin virheellinen tutkimus on aika tavallista. Se keskustelu mitä olen nähnyt on johtunut siitä, että tutkimus on nostettu julkisuuteen, ei sen tieteellisistä ansioista.

      https://www.avaruus.fi/uutiset/kosmologia-ja-teoreettinen-fysiikka/mullistava-vaite-mustat-aukot-ovat-vastuussa-pimeasta-energiasta.html

  4. Jos mustat aukot kasvaisivat itsestään maailmankaikkeuden skaalatekijän kolmannessa potensissa, silloin ne olisivat (jos laskin oikein) kolminkertaistuneet massaltaan 5 miljardissa vuodessa ja 30-kertaistuneet 10 miljardissa. Vanhoja tähtiä on galaksissamme melko paljon, joten myös vanhoja tähdenmassaisia aukkoja pitäisi olla paljon. Jos väitetään että aukot kasvavat tuolla tavalla oudosti, niin Linnunradan tapaisissa galakseissa pitäisi olla yli tähdenmassaisia muutaman sadan tai tuhannen auringonmassan aukkoja aika paljon, ehkä jopa enemmän kuin nuoria tähdenmassaisia, koska Linnunradan tähtituotto on nykyään pienempi kuin se oli galaksin nuoruudessa. Luulisi että tuollainen näkyisi mm. gravitaatioaaltotapahtumien tilastoissa. Perinteisesti on ihmetelty pikemminkin keskiraskaiden aukkojen vähäisyyttä datoissa, eikä yliedustusta.

    1. Syksy Räsänen sanoo:

      Ensiksihän on todettava, että kirjoittajien ehdottamaa kasvumekanismia ei ole olemassa. Mustien aukkojen kasvun reunaehdot laajenevassa maailmankaikkeudessa tunnetaan yleisen suhteellisuusteorian simulaatioista, eikä niihin liity mitään mysteeriä, toisin kuin mitä kirjoittajat väittävät.

      Toisekseen isojen mustien aukkojen energiatiheys on paljon pienempi kuin pimeän energian energiatiheys. Sille on tiukat rajat havainnoista.

      Ja niin edelleen.

      1. Eusa sanoo:

        Tai sitten pimeän energian energiataseen tulisi olla jotain aivan muuta kuin miten sen yleensä ymmärretään toimivan. Tutkimuksessa ei anneta mitään mielekästä käsittelyä tuolle vaatimukselle, joten on ad hoc -nosto eikä jatkoon.

  5. Kari Tanner sanoo:

    Hyvä kirjoitus jälleen.

    Tieteellisten tutkimuksien uutisointi tavallisissa lehdissä on tosiaan ongelmallista ihan näin maalikonkin näkökulmasta. Usein se ”totuus” unohtuu kaiken ”spekulaation ja villin ideoinnin” alle, enkä tosian puhu pelkästä fysiikasta.

    Sinällään ideointi ja revittely kuuluu fysiikaan ( mahdollisuus uuden löytymiseen) mutta mikä on kokonaiskuva ja relevanttia onkin toinen asia.

    Osin syy on varmasti lehdistön: Ei uutta standardimallissa vs. Pimeä aine on selitetty. Kumpi myy:) 🙂

    1. Cargo sanoo:

      Muistan joskus lukeneeni, miten Einsteinin suhteellisuusteoriasta uutisoitiin keisarillisen ajan Suomessa, ja se meni jotakuinkin näin: ”Uuden selityksen mukaan permannolle heitetyt kellot käyvät kaikki eri aikaa.” 🙂

      1. Aivan, hyvä muistutus siitä että tiedotusvälineet ovat epäonnistuneet ennenkin, eikä se ole estänyt tiedettä saamasta tuloksia. Mutta nykyajan ongelma on että osa tutkijoista tavoittelee julkisuutta eikä keskity omaan tekemiseensä. Ehkä jopa väittävät että systeemi pakottaa.

        Vaikka tutkijoita on Einsteinin aikaan verrattuna enemmän, merkittäviä keksintöjä tehdään samaa tahtia kuin aiempina vuosisatoina, eli 30-100 vuoden välein. Vähän tiheämmin kuin Linnunradassa räjähtää supernovia.

  6. Kari Ojala sanoo:

    Seuraava Sabine Hossenfelderin (Ph.D, teor. fyysikko) tuore youtube-video nostaa erään ison ja mielenkiintoisen kissan pöydälle:
    What’s Going Wrong in Particle Physics? (This is why I lost faith in science.)
    https://www.youtube.com/watch?v=lu4mH3Hmw2o

    Koska video ainakin osittain liittyy Syksyn teemaan, olisin kiinnostunut kuulemaan ajatuksia koskien Sabinen vlogia, jos ei tässä ketjussa niin kenties erillisen blogin yhteydessä. Aihehan on laaja ja ansaitsee ehkä ihan oman bloginsa Syksyltä.

    1. Syksy Räsänen sanoo:

      Sabine Hossenfelderillä on sekä aiheellista että aiheetonta kritiikkiä hiukkasfysiikkaa kohtaan (mutta yleisesti ottaen hyvä, että hänenlaisiaan kriitikoita on). En yleensä jaksa katsoa videoita, mutta voisin ehkä kommentoida hänen arvosteluaan jossain vaiheessa. Pannaan mietintään.

  7. Jani sanoo:

    Toinen asia mistä olisi kiva kuulla Syksyn kommentit on nämä tiedelehdet. Näitähän on jokunen kun mainitusta TM:stä ja hesaristakin tiedeosio löytyy. Mitkä olisivat suomenkielinen ja englanninkielinen tiedelehti jotka kannattaisi tilata?

    1. Syksy Räsänen sanoo:

      Tähdet ja avaruus on tähtitieteen alalta hyvä lehti.

      Englanninkielisistä suosittelen lehtiä Quanta ja Aeon.

      https://www.quantamagazine.org/

      https://aeon.co/science

  8. Martti V sanoo:

    Ilmeisesti on epäselvää miten mustat aukot kasvoivat niin nopeasti kosmoksen syntyaikoina. JWTS uusimmat havainnot kielii, että galaksit kehtittyivät myös nopeasti. Kenties pimeäenergia oli aiemmin vähemmän.

  9. Erkki Kolehmainen sanoo:

    Pekka Janhusen kommentti oli hyvä: ”Aivan, hyvä muistutus siitä että tiedotusvälineet ovat epäonnistuneet ennenkin, eikä se ole estänyt tiedettä saamasta tuloksia. Mutta nykyajan ongelma on että osa tutkijoista tavoittelee julkisuutta eikä keskity omaan tekemiseensä. Ehkä jopa väittävät että systeemi pakottaa.”

    Tämä pitää paikkansa muunkin kuin tieteen suhteen. Esim. Sanoma Median IS levittää pelkästään Ukrainan sotapropagandaa antamatta millekään muulle sijaa!

    Tieteen popularisointi on vaikeaa. Ihmiset eivät tunne tieteen terminologiaa eivätkä edes perusteita. Oma väitöskirjani käsitteli sappihappojen solubilisaatio-ominaisuuksia. Tein siitä A4-kokoisen lehdistötiedotteen. Savon Sanomat julkaise sen sellaisenaan. Hesarin toimittaja päätti editoida minun tekstiäni sillä seurauksella, että 24 hiiliatomia sisältävistä sappihapoista tuli sinihappoa HCN! Hesari ei korjannut virhettään, vaikka pyysin sitä. Janne Virkkunen oli silloin Hesarin päätoimittaja.

    Yleisesti ottaen jonkin teorian oikeaksi todistaminen on vaikeaa. Vaikka teoria antaisi oikean tuloksen kaikissa tunnetuissa tapauksissa, se ei takaa, että näin olisi jatkossa. Newtonin mekaniikalla selvittiin aika kauan, mutta lopulta suhteellisuusteoria syrjäytti sen tai Newtonin mekaniikka sisältyy uuteen teoriaan rajatapauksena. Teorian vääräksi osoittamiseen tarvitaan vain yksi koe!

    1. Syksy Räsänen sanoo:

      Jätetään keskustelu sotauutisoinnista muille foorumeille.

      Teorioiden osoittaminen vääriksi havainnoilla on itse asiassa hieman monimutkaisempaa, ks. https://www.ursa.fi/blogi/kosmokseen-kirjoitettua/edistys-ja-rappio/

Vastaa

Sähköpostiosoitettasi ei julkaista. Pakolliset kentät on merkitty *


Palikkatesti

30.1.2023 klo 22.51, kirjoittaja
Kategoriat: Kosmokseen kirjoitettua

Kirjoitin viime kuussa siitä, miten Gerardus ‘t Hooft ja Martinus J. G. Veltman vuonna 1971 osoittivat, että hiukkasfysiikan Standardimallissa on vain äärellinen määrä erilaisia vuorovaikutuksia hiukkasten välillä. Tämän läpimurron takia Standardimallista voi luotettavasti ennustaa havaintoja ilman lisäoletuksia.

Standardimalli kuvaa kaikkea tunnettua fysiikkaa gravitaatiota lukuun ottamatta, joten tulos oli hyvin merkittävä. Todistuksessa käytetty menetelmä, renormalisaatio, on myös suuresti johdattanut hiukkasfysiikkaa vuosikymmenien ajan, mutta ei aina hedelmälliseen suuntaan.

Kun kvanttikenttäteoriaa lähdetään rakentamaan, siinä on yleensä vain pieni määrä erilaisia vuorovaikutuksia kenttien välillä. Esimerkiksi sähkömagnetismin tapauksessa niitä on vain yksi: sellainen missä elektroni lähettää fotonin.

Erilaisia tapahtumia voidaan selittää kasaamalla yhdestä palikasta isompia kokonaisuuksia. Jos sen kääntää ajassa ympäri, niin saa vuorovaikutuksen, missä elektroni vastaanottaa fotonin. Jos tämän laittaa yhteen edellisen palikan kanssa, tuloksena on tapahtuma, missä elektronit vuorovaikuttavat toistensa kanssa vaihtamalla fotonin. Tämä kuvaa sitä, miten sähkövaraukset hylkivät toisiaan. Laittamalla samat kaksi palikkaa hieman eri järjestykseen saa tapahtuman, missä elektroni ja sen antihiukkanen positroni annihiloituvat, eli häviävät ja synnyttävät kaksi fotonia.

Sähkövarausten hylkiminen ja aineen ja antiaineen annihilaatio vaikuttavat aivan erilaisilta ilmiöiltä, mutta kvanttikenttäteoriassa ne ovat samanlaisia yksinkertaisia seurauksia sähkömagnetismin perusvuorovaikutuksesta.

Mitä tarkemmin kvanttiefektejä laskee, sitä useampia uudenlaiselta näyttäviä palikoita ne tuovat mukaan teoriaan. Jos teoria on renormalisoituva, nämä kaikki vuorovaikutukset voidaan kuitenkin esittää alkuperäisten palikkojen avulla.

Esimerkiksi sähkömagnetismissa alkuperäinen vuorovaikutus, missä elektroni lähettää fotonin, näyttää yksinkertaiselta vain jos sitä katsoo karkeasti. Syvemmälle syynätessä näkyy, että karkeampi kuva koostuu pienemmistä osasista, joissa on sitä enemmän palikoita mitä tarkemmin katsoo. Elektronin sähkövaraus (eli se, miten vahvasti se kytkeytyy fotoniin) riippuu fotonin energiasta, mutta vuorovaikutus säilyy muuten samanlaisena. Renormalisoituvassa teoriassa tarkemmat laskut paljastavat vain hienompaa rakennetta alkuperäisissä vuorovaikutuksissa, eivät tuo mukaan muuta uutta.

Kvanttikorjauksissa kaikki kentät vuorovaikuttavat toisiinsa. Standardimallissa Higgsin hiukkasen (ja vain Higgsin hiukkasen) massa on herkkä näille kvanttikorjauksille. Niiden kautta isomassaiset hiukkaset tekevät myös Higgsistä raskaan: Higgsin massa on suunnilleen yhtä iso kuin raskaimman hiukkasen massa.

Standardimallissa näin on. Higgs on suunnilleen yhtä raskas kuin top-kvarkki, raskain tunnettu hiukkanen. Mutta jos raskaampia hiukkasia on olemassa, miksi Higgsin hiukkasen massa on pienempi kuin niiden? Tämä kysymys tunnetaan nimellä hierarkiaongelma, ja siitä on kirjoitettu satoja tai tuhansia tieteellisiä artikkeleita.

Hierarkiaongelman taustalla on se, että 1970-luvulla ajateltiin, että Standardimalli on osa suurta yhtenäisteoriaa. Suuren yhtenäisteorian toistaiseksi tuntemattomien hiukkasten pitää olla paljon raskaampia kuin tunnettujen, koska muuten niistä olisi nähty merkkejä.

Tämä johdatti supersymmetriana tunnetun idean soveltamiseen hiukkasfysiikkaan. Supersymmetria katkaisee Higgsin riippuvuuden raskaampien hiukkasten massoista. Toinen yritys oli tekniväri, missä Higgs ei ole alkeishiukkanen. Silloin Higgsin massa (kuten kvarkeista ja gluoneista koostuvan protonin massa) määräytyy siitä millaisia osia siinä on ja miten ne vuorovaikuttavat, eivätkä raskaammat hiukkaset vaikuta siihen.

Yhteistä molemmille selityksille on se, että Higgsin massaa vastaavilla energioilla pitäisi näkyä uutta fysiikkaa, joka muuttaa Higgsin käytöstä siitä mitä Standardimallin ennustaa.

Nyt LHChiukkaskiihdyttimessä on luodattu energioita, jotka ovat Higgsin massaa kymmenen kertaa isompia, eikä supersymmetriasta, tekniväristä tai muista Standardimallin laajennuksista ole näkynyt merkkiäkään. Niinpä yhä useampi fyysikko saattaa olla valmis ratkaisemaan ongelman yksinkertaisella tavalla: ehkä Higgs ei ole raskaampi siksi, että raskaampia hiukkasia ei ole olemassa.

Raskaampia hiukkasia ei välttämättä tarvita kosmologian neljän suuren avoimen ongelman ratkaisemiseen. Vuodesta 2007 alkaen on hahmotettu, että kosmisen inflaation voi hoitaa Higgsillä. Pimeä aine sekä aineen ja antiaineen epäsuhta voidaan selittää uusilla hiukkasilla, jotka ovat Higsin hiukkasta kevyempiä, erimerkiksi aksioneilla tai oikeakätisillä neutriinoilla kuten kauniissa NUMSMmallissa. Neljättä ongelmaa, kiihtyvää laajenemista, ei taasen yleensä edes yritetä selittää raskailla hiukkasilla.

Tässä on se hyvä puoli, että kevyempiä hiukkasia voi olla helpompi havaita kuin raskaita, koska niiden tuottamiseen tarvitaan vähemmän energiaa. Havaitsemista toisaalta vaikeuttaa se, että monet ehdotetut uudet kevyet hiukkaset (kuten aksionit ja oikeakätiset neutriinot) vuorovaikuttavat hyvin heikosti. Osa teoreetikoista ja kokeilijoista onkin kääntynyt kaavailemaan energioiden kasvattamisen sijaan törmäysten määrän nostamista kiihdyttimissä heiveröisten vuorovaikutusten erottamiseksi, kuten ehdotetussa CERNin SHIP-kokeessa.

29 kommenttia “Palikkatesti”

  1. Lentotaidoton sanoo:

    ”Osa teoreetikoista ja kokeilijoista onkin kääntynyt kaavailemaan energioiden kasvattamisen sijaan törmäysten määrän nostamista kiihdyttimissä heiveröisten vuorovaikutusten erottamiseksi, kuten ehdotetussa CERNin SHIP-kokeessa.”

    Mielenkiintoinen koe, mutta muutama vuosi saadaan vielä odotella:
    The construction and installation will last until the third long shutdown of the LHC and the data taking is assumed to start in 2026.
    https://iopscience.iop.org/article/10.1088/1742-6596/878/1/012014/pdf
    myös täällä:
    https://arxiv.org/pdf/2112.01487.pdf

  2. Cargo sanoo:

    ”Jos teoria on renormalisoituva, nämä kaikki vuorovaikutukset voidaan kuitenkin esittää alkuperäisten palikkojen avulla.”

    Tuli mieleen, että voisiko tuota palikoiden ynnäämistä verrata optiikan aalto-oppiin ja Huygensin periaatteeseen, jossa jokainen aaltorintaman piste toimii uutena aaltoilun lähteenä? Huygensin periaate selittää ilmiöitä, mutta sen epäfysikaalinen idea laskea kaikki luvuttomat säteilylähteet yhteen johtaa ymmärtääkseni äärettömiin tuloksiin. Lopulta kai Kramers selitti valon dispersion Fourierin menetelmien avulla – ja Heisenberg nappasi siitä idea kvanttimekaniikkaansa kuvaamaan hiukkasen olemattoman liikeradan paikkaa ja liikemäärää käänteisten Fourierin sarjojen avulla. Voisiko siis tuo kvanttikenttäteorian palikkalaskenta vastata jotakin sopivaa muunnosta, joka sitten selittää havainnot ilman approksimaatioita?

    1. Syksy Räsänen sanoo:

      Ei, renormalisaatio on monimutkaisempi asia.

  3. Erkki Kolehmainen sanoo:

    ”…tuloksena on tapahtuma, missä elektronit vuorovaikuttavat toistensa kanssa vaihtamalla fotonin. Tämä kuvaa sitä, miten sähkövaraukset hylkivät toisiaan.” Järkeenkäypä selitys. Entä jos elektroni on sidottu atomiin tai molekyylin kovalenttiseen sidokseen? Sepä ei enää hyväksyykään mitä tahansa fotonia vaan sellaisen, jonka energia vastaa elektronin sallittujen energiatilojen erotusta. Kuinka suuri osa maailman kaikista elektroneista on vapaita ja kuinka suuri osa sidottuja? Kyse on siis elektronien demokratiasta!

    1. Syksy Räsänen sanoo:

      Vain pieni osa aineesta on muodostanut tähtiä tai planeettoja, suurin osa on yhä kaasuna avaruudessa. Tähdistä tuleva valo on rikkonut suurimman osan kaasusta atomit siten, että elektronit ovat irtonaisina. Suurin osaa maailmankaikkeuden elektroneista on siis vapaita. (En nyt osaa sanoa tarkkaa lukumäärää.)

      1. Lentotaidoton sanoo:

        Niin maailmankaikkeuden koko energiabudjetista vain noin 5% on (tavallista) baryonista ainetta. Ja siitäkin galaksien tähtien osuus on vain n 7%. Loppu on kylmää/lämmistä/kuumaa kaasua galakseissa ja galaksienvälisessä avaruudessa.
        https://sci.esa.int/web/xmm-newton/-/60430-the-cosmic-budget-of-ordinary-matter

  4. Eusa sanoo:

    t’Hooftin nimi kirjoitettu väärin.

    Mitä mieltä olet hänen viimeaikaisista mustan aukon tutkimuksistaan, joissa ollaan päätymässä siihen, ettei tapahtumahorisontin sisäpuolta ole tai se on epäonnistunut käsite?

    1. Syksy Räsänen sanoo:

      Millä tavalla väärin? Oikea muoto on ’t Hooft, ei t ’Hooft.

      En ole lukenut noita papereita. En muutenkaan seuraa tutkimusta mustien aukkojen ja kvanttifysiikan yhteensovittamisesta.

      1. Eusa sanoo:

        Hups. Muistin heittopilkun paikan tosiaan itse väärin. Viestivaihdossa näkyy jatkuvasti ilman pilkkua ”t Hooft”.

        Mustan aukon tutkimus linkittyy energiajakaumamuutosten tilassa eli gravitaation kvantittamiseen ja kvanttimekaniikan yhteensovittamiseen yleisen suhteellisuuden kanssa.

        1. Eusa sanoo:

          Siis suosittelen kyllä seuraamaan tuota tutkimuslinjaa.

      2. Lentotaidoton sanoo:

        Niin nämä Hollantilaiset nimet on hauskoja. Jos tämän fyysikon nimellä (ilman etunimeä) aloittaa lauseen niin ensimmäisenä tulee yläheittomerkki. Esim näin:
        ’t Hooft is most famous for his contributions to the development of gauge theories in particle physics.

  5. taas se meni venäjille sanoo:

    tarvitaanko singulariteettia edes? käsite ”ääretön tiheys” on järjenvastainen; auringosta tulee musta aukko jos se puristuu 3km mittaiseksi eikä nollatilavuuteen

    1. Syksy Räsänen sanoo:

      Yleisen suhteellisuusteorian mukaan aineen romahtaessa mustan aukon sisällä syntyy singulariteetti, jossa tiheys kasvaa rajatta eikä yleinen suhteellisuusteoria enää päde.

      Mutta koska kvanttifysiikkaa ja yleistän suhteellisuusteoriaa ei ole saatu kunnolla sovitettua yhteen, eo olla varmoja siitä, mitä mustien aukkojen sisällä tapahtuu.

      Kun kysymys ei liity merkinnän aiheeseen, niin ei siitä sen enempää.

  6. Eusa sanoo:

    Mikä puoltaisi sitä, että oikeakätisyys tekisi neutronista hiukkasen omillaan eikä pelkästään antihiukkasta?

    1. Syksy Räsänen sanoo:

      Kyse on neutriinoista, ei neutroneista. Tarkemmin, ks. https://www.ursa.fi/blogi/kosmokseen-kirjoitettua/kauneusvirheen-korjaaminen/

      1. Eusa sanoo:

        Totta kai neutriinoista kyse, typo-erhe.

        Linkkaamassasi artikkelissa et käsitellyt antineutriinoja.

        Onko niin, että jos neutriino lopulta osoittautuisi Majorana-hiukkaseksi ollen helisiteettinsä perusteella hiukkanen tai antihiukkanen, välttämättä se varmistuisi massattomaksi ja makuoskillaatioteoria menisi romukoppaan?

        Toinen yleisesti hellitty idea symmetriarikosta neutriinoissa ja sitä kautta aine-antiaine-epätasapainon selitysmalli taitaisi myös olla vaikeuksissa mikäli noin osoittautuisi olevan.

        Onko tiedossasi mitään vakavaa tutkimusta tuollaisella skenaariolla?

        1. Syksy Räsänen sanoo:

          Majorana-neutriinoilla voi olla massa. Itse asiassa useimmissa neutriinojen massoja selittävissä malleissa neutriinot ovat Majorana-neutriinoja.

          1. Eusa sanoo:

            Silloin massallisuus hoituu seesaw-mekanismin massamatriisilla Majorana-vaihein, mikä ei nyt aivan ”rehellistä” ole.

            Massattomuusolettama on havainnoista päätellen kuitenkin varsin vahvoilla, sillä neutriinot vaikuttavat liikkuvan aina nopeudella c oikeakätisellä helisiteetillä ja antineutriinot aina nopeudella c vasenkätisellä helisiteetillä. Massalliselle hiukkaselle luontaista kiraliteetin invarianttia rakennetta ei ole todennettu.

            Edelleen olisin kiinnostunut tutkimuksesta, jossa lähtökohtana neutriinojen aito massattomuus ja silti makujen sekoittuminen etenemisessä.

          2. Syksy Räsänen sanoo:

            Ei nyt tässä neutriinoista sen enempää, kun ovat sivuseikka merkinnässä.

  7. Cargo sanoo:

    ”Erilaisia tapahtumia voidaan selittää kasaamalla yhdestä palikasta isompia kokonaisuuksia. Jos sen kääntää ajassa ympäri, niin saa vuorovaikutuksen, missä elektroni vastaanottaa fotonin. Jos tämän laittaa yhteen edellisen palikan kanssa, tuloksena on tapahtuma, missä elektronit vuorovaikuttavat toistensa kanssa vaihtamalla fotonin. Tämä kuvaa sitä, miten sähkövaraukset hylkivät toisiaan. Laittamalla samat kaksi palikkaa hieman eri järjestykseen saa tapahtuman, missä elektroni ja sen antihiukkanen positroni annihiloituvat, eli häviävät ja synnyttävät kaksi fotonia.”

    Voiko aika kulkea noiden vuorovaikutusten ja alkutilojen suhteen molempiin suuntiin? Meinaan vaan, että miksi kahden fotonin pitäisi ylipäätään vuorovaikuttaa keskenään ja muodostaa hiukkaisia, ja juuri elektroni-positroni-parin juuri samoilla nopeuksilla. Jos tämä käytönnön mahdottomuus ilmenee myriadeissa vuorovaikutustilanteissa (jotka vieläpä virtuaalisia?), niin miten ajan suuntaa voitaisiin kääntää edes periaatteessa? Mielestäni aika on pohjimmiltaan tilastollinen käsite, jonka vieminen kvanttivärinän paikallistasolle on vain filosofien kuumeista houreilua.

    1. Syksy Räsänen sanoo:

      En ole varma ymmärränkö kysymystä. Kaikki hiukkaset vuorovaikuttavat jollain tavalla muiden hiukkasten kanssa. Teoria kertoo todennäköisyyden sille, vaikuttavatko ne tietyssä tilanteessa ja millä tavalla.

      Siitä mitä tiedämme ajasta, ks. https://www.ursa.fi/blogi/kosmokseen-kirjoitettua/kaksi-tarinaa-ajasta/ (Ei siitä sen enempää, koska ei liity merkinnän aiheeseen.)

      1. Cargo sanoo:

        Onko tuo kahdesta fotonista syntyvä elektroni-positroni-pari kuinka spontaani prosessi? Jos ”sallitussa” prosessissa entropia kasvaa ja energia leviää, niin mikähän tuollaisen aineenmuodostuksen voisi saada aikaan. Esimerkiksi kvanttivärinän stimuloima fotonin emissio vetyatomissa on selvästi spontaani prosessi, sillä säteilyenergiaa leviää ja elektroni siirtyy alemmalle energiatasolle. (Tämä varmaankin liittyy ajan nuoleen, mutta siitä en nyt tohdi kysyä, koska ei liity merkinnän aiheeseen.)

        1. Syksy Räsänen sanoo:

          Kun kaksi hiukkasta -olivatpa ne fotoneita tai protoneita- kohtaavat, niillä on todennäköisyys vuorovaikuttaa keskenään. Kvanttikenttäteoria kertoo, mitkä ovat erilaisten todennäköisyyksien mahdollisuudet, esimerkiksi että fotonit muuttuvat elektroneiksi tai joiksikin muiksi hiukkasiksi. Tähän ei tarvita mitään hiukkasista ylimääräistä. Termiä spontaani ei kuitenkaan käytetä tässä yhteydessä.

  8. Eusa sanoo:

    Mitähän blogi-isäntä sekoilee? Kaksi merkityksetöntä kommenttimerkintää… ?

    1. Syksy Räsänen sanoo:

      Pahoitteluni, wifi saattoi takkuilla. Poistin tyhjät kommentit nyt.

  9. Martti V sanoo:

    Vaikka higgs vastaisi inflaatiosta se ei tarkoita, ettei ennen inflaatiota olisi ollut raskaampia hiukkasia. Hiukkaskokeet ovat niin kaukana vaaditusta energiaskaalasta, ettei voida vetää johtopäätöksiä.. Sen sijaan standardimallista poikkeavia ilmiöitä on viime aikoina havaittu (esim Beta hajoamis kokeet) – toki poikkeavuudet ovat odotetusti hyvin pieniä.

    1. Syksy Räsänen sanoo:

      Oleellista on se, kuinka isoja poikkeamat ovat suhteessa tilastollisiin ja systemaattisiin virheisiin. Tällä hetkellä maanpäällisissiä kokeissa ei ole havaittu mitään poikkeamia Standardimallista (neutriinojen massoja lukuun ottamatta), jotka olisivat tilastollisesti merkittäviä ja joissa systemaattiset virheet ovat pieniä (mukaan lukien epävarmuudet teoreettisissa ennusteissa).

      1. Lentotaidoton sanoo:

        On ymmärrettävää, että Standardimallin tuonpuoleista fysiikkaa haetaan kissoin koirin (kun tiedetään osin Standarditeorian ilmeiset puutteet). Ja tiedetään myös tämänhetkinen kiihdyttimien (esim LHC) suhteellisen pienet energiat. Tässä suurimmat pettymykset ehkä on koettu supersymmetrian osalta. On kuitenkin ilmeinen totuus, että energian ”korpimailla” Planckin energiaan täytyy tulla vastaan uutta teoriaa (ja uusia hiukkasia).

        Tästä tilanteesta ja tilanteessa uudisnälkäinen media yrittää joskus väkisin vääntää ”sensaatioita”. Osittain hyvä, osittain huono asia. Tuo Syksyn mainitsema tilastollisen poikkeavuuden merkitävyys vain joskus räikeästi unohtuu tiede-otsikoinnissa.

Vastaa

Sähköpostiosoitettasi ei julkaista. Pakolliset kentät on merkitty *

Sulka ja vasara

24.1.2023 klo 19.53, kirjoittaja
Kategoriat: Kosmokseen kirjoitettua

Toinen elokuuta vuonna 1971 David Scott pudotti oikeasta kädestään haukansulan ja vasemmasta vasaran. Koska hän oli Kuussa, missä ei ole ilmakehää, ne osuivat kamaraan yhtä aikaa. Nauhoitus on katsottavissa avaruusjärjestö NASAn YouTube-kanavalla. Siinä Scott sanoo leikillisesti, että koska Galileo Galilei on yksi syy siihen, että hän oli Kuussa, se on hyvä paikka varmistaa tämän teoria siitä, että kaikki kappaleet putoavat samaa tahtia niiden koostumuksesta riippumatta.

Galilei oli pohtinut 1600-luvulla kappaleiden pudottamista Pisan kaltevasta tornista, mutta ei ilmeisesti koskaan tehnyt sellaista koetta. Sen sijaan hän vertasi erimassaisia heilureita ja totesi, että ne kaikki liikkuvat samalla tavalla, noin prosentin tarkkuudella.

Isaac Newtonin gravitaatioteoria selitti ilmiön vuonna 1686. Sen mukaan kappaleen kiihtyvyys on verrannollinen siihen kohdistuvaan voimaan jaettuna kappaleen massalla, ja gravitaatiovoima on verrannollinen kappaleen massaan. Massiivisempiin kappaleisiin kohdistuu isompi voima, mutta niitä pitää myös puskea enemmän, joten kappaleet liikkuvat samalla tavalla massasta riippumatta.

Selitys avaa kuitenkin heti uuden kysymyksen. Painava massa (joka määrää gravitaation voimakkuuden) ja hitausmassa (joka kertoo miten paljon kappaleita pitää työntää) ovat aivan erilaisia asioita. Esimerkiksi sähkövoima on verrannollinen sähkövaraukseen massan sijaan, joten eri kappaleet liikkuvat eri tavalla sähkökentässä. Miksi gravitaatiovaraus eli painava massa sen sijaan liittyy hitauteen?

1800-luvun lopulle tultaessa fyysikko Loránd Eötvös oli kasvattanut painavan massa ja hitausmassan eron mittauksen tarkkuutta prosentin miljoonasosaan. Kehittäessään yleistä suhteellisuusteoriaa vuodesta 1907 alkaen Albert Einstein otti yhdeksi lähtökohdaksi tämän oudon yhteensattuman massojen välillä. Hän arveli, että gravitaatio ja hitaus liittyvät toisiinsa siksi, että gravitaatiossa on kyse aika-avaruuden ominaisuuksista.

Idea vei Einsteinin oikealle polulle, ja yleinen suhteellisuusteoria lopulta selitti asian tyydyttävästi. Sen mukaan gravitaatio ei ole voima, vaan aika-avaruuden kaarevuuden ilmentymä. Kappaleet joihin ei vaikuta voimia liikkuvat suoraa reittiä kaarevassa aika-avaruudessa. Suorat reitit ovat samoja kaikille, ne eivät riipu siitä kuka niitä kulkee.

Tapaus havainnollistaa sitä, että joskus ratkaiseva vihje oikean teorian löytämiselle fysiikassa ei liity ristiriitaan havaintojen ja teorian välillä. Painavan massan ja gravitaatiomassan läheisyys 1800-luvun lopulla ei ollut ristiriidassa minkään teorian tai muun havainnon kanssa. Empiirisyydessä on kyse paljon muustakin kuin teorioiden ennusteiden vertaamisesta havaintoihin. Joskus kokeet jotka eivät löydä mitään ovat yhtä tärkeitä kuin kokeet, jotka paljastavat jotain uutta.

Sitä putoavatko kappaleet samalla tavalla on sitten Einsteinin päivien mitattu monin tavoin. Yksi keino on verrata Maan ja Kuun liikettä Auringon ympäri. Jos ne eivät putoa (eli kierrä) samaa tahtia, niin Maan ja Kuun etäisyys toisistaan muuttuu ajan myötä. Apollo 15 -lento, jossa Scott oli mukana, jätti Kuuhun peilin, joka heijastaa valonsäteet takaisin niiden tulosuuntaan. Myös lennot Apollo 11 ja 14 sekä Lunokhod 1 ja 2 veivät vuosina 1969-73 tällaisia peilejä Kuuhun. Mittaamalla lasersäteen matka-ajan Maasta Kuussa olevaan peiliin ja takaisin niiden etäisyyden muutosta voidaan seurata millimetrien tarkkuudella. Kokeiden mukaan Kuu ja Maa kiertävät Aurinkoa samalla tavalla ainakin kymmenestuhannesosan miljardisosan tarkkuudella.

Tiukimman rajan on antanut vuonna 2016 laukaistu MICROSCOPE-satelliitti, jonka tulokset julkaistiin viime syyskuussa. MICROSCOPEn sisällä oli kaksi eri metalleista valmistettua sisäkkäistä sylinteriä, ja kokeessa seurattiin liikkuvatko ne toistensa suhteen. Kuten Kuussa, kiertoradalla on rauhallisempaa kuin kaikenlaisista häiriöistä kärsivällä Maapallolla. Koeryhmä totesi, että kappaleet putoavat samalla tavalla miljoonasosan miljardisosan tarkkuudella.

Nykyään näitä kokeita tehdään juuri siksi, että yleisen suhteellisuusteorian mukaan mitään ei pitäisi näkyä. Samaa tahtia putoaminen on herkkä testi siitä, minne yleisen suhteellisuusteorian pätevyysalue ulottuu. On satoja laajennuksia yleiselle suhteellisuusteorialle, ja monissa niistä kappaleet putoavat hieman eri tavalla. Yli sadan vuoden ajan yleinen suhteellisuusteoria on selvinnyt kaikista kokeista, mutta koetus jatkuu.

35 kommenttia “Sulka ja vasara”

  1. Antti sanoo:

    tippuuko ne myös kvanttimaailman kokoluokassa samaan aikaan vai onko siinä joku

    vähimmäiskoko johon kokeen rajat tulevat vastaan?

    1. Syksy Räsänen sanoo:

      Hyvä kysymys. Yksittäisten hiukkasten, jotka ovat tilassa jossa kvanttiefektit ovat merkittäviä, putoamista gravitaatiokentässä on mitattu. Tuloksena on, että ne putoavat samaa tahtia. Teoreettisesti jossain vaiheessa myös gravitaatiota pitäisi käsitellä kvanttifysiikan mukaisesti, mutta se lienee vielä kokeista kaukana. (Lukuun ottamatta kosmista inflaatiota varhaisessa maailmankaikkeudessa.)

  2. Rotannahka sanoo:

    painovoimalain toimivuudesta erittäin pienillä etäisyyksillä;
    mikäli teemme reiän omenan läpi, ja muurahainen kävelee tämän kautta omenan keskiöön – eikö Newtonin lain m1 x m2 / r^2 mukaan painovoiman pitäisi lähestyä ääretöntä välimatkan lähestyessä nollaa?

    kiitos!

    1. Syksy Räsänen sanoo:

      Ei, koska massa menee nollaan.

      1. Rotannahka sanoo:

        Miksi maa vetää muurahaista yhä puoleensa, jos massa on nolla?

        1. Syksy Räsänen sanoo:

          Muurahaisen massa ei ole nolla.

          1. Rotannahka sanoo:

            Juurihan Te sanoitte massan ”menevän nollaan”?
            käsitinkö taas väärin

          2. Syksy Räsänen sanoo:

            Tarkoitin muurahaista puoleensa vetävää massaa, en muurahaisen massaa. Kun muurahainen lähestyy omenan keskipistettä, muurahaisen ja keskipisteen välissä oleva massan määrä lähestyy nollaa, koska muurahaisen ja keskipisteen välissä oleva tilavuus lähestyy nollaa.

          3. Rotannahka sanoo:

            Eikö kaavassa lasketa nimenomaan muurahaisen & omenan massa – eikä näiden välistä massaa?
            Entä omenan keskiössä oleva ilma, meneekö tämännkin massa nollaan?

          4. Syksy Räsänen sanoo:

            Jos aineen tiheys on vakio, tilavuuden V sisältämä massa on tiheys kertaa V. Pallosymmetrisessä tapauksessa tilavuus on 4 pi/3 kertaa r^3, missä r on pallon säde. Pallosymmetrisen massan gravitaatiovoima Newtonin teoriassa on verrannollinen massaan per säde^2, toisin sanoen r^3/r^2=r. Gravitaatovoima on siis sitä pienempi, mitä lähempänä keskustaa ollaan.

            Tämä riittäköön tästä.

  3. Erkki Kolehmainen sanoo:

    Jos tuon kuussa demonstroidun kokeen tekijä olisi ollut neuvostoliittolainen, niin silloin olisi pudotettu sirppi ja vasara. Toki sulka ja vasara ovat parempi pari ilmanvastuksen vaikutuksen osoittamiseksi! Kokeiden tekemisessä on kaksi optiota. Ensinnä voi yrittää tehdä koe tarkemmin ja paremmin kuin ennen. Esim. Syksyn kuvaaman ”tonni tankissa” kokeen Xe-pöntön kokoa voidaan kasvattaa. jolloin halutun havainnon todennäköisyys kasvaa. Ei kovin luovaa ajattelua. Toinen tapa on suunnitella kokokaan uusi koe, mikä vaatii innovaatiokykyä ja uutta teoreettista ajattelua. Aika usein mennään tuon ensiksi mainitun kaavan mukaan eli ainoa innovaatio on laitteen koon kasvattaminen, kunnes taloudelliset realiteetit lopettavat käytännössä laitteen kehittämisen.

  4. Cargo sanoo:

    ”Painava massa (joka määrää gravitaation voimakkuuden) ja hitausmassa (joka kertoo miten paljon kappaleita pitää työntää) ovat aivan erilaisia asioita.”

    Kun Newton päätteli F=ma, niin eikö hän käyttänyt painovoimaa apunaan? Meinaan vaan, että ehkei hitausmassaa ole määritelty riippumattomalla tavalla painavaan massaan nähden.

    1. Syksy Räsänen sanoo:

      En tiedä miten historiallisesti Newtonin päätyi lakiin F=ma. Toki hän gravitaatiolaissaan oletti, että painava massa on sama kuin hitausmassa. Tämä ei muuta sitä, että ne ovat eri käsitteitä, eikä hänen teoriansa selitä sitä, miksi ne ovat yhtäsuuria.

      1. Cargo sanoo:

        Jos kappale on käytännössä lokalisoitunut klimppi sidosenergiaa, niin eikö kaavan E = mc^2 kautta klassisen liikkeen tarkastelu siirry painavan massan aiheuttaman liikkuvan kaareutuman tarkasteluun?

        1. Syksy Räsänen sanoo:

          En ole varma ymmärränkö kysymystä. Aine ja aika-avaruus ovat erillisiä asioita. Kappaleet eivät ole osa aika-avaruutta, ne liikkuvat siinä ja kaareuttavat sitä.

          1. Cargo sanoo:

            Koitin kait puntaroida, että hitaus- ja painomassaan pitäisi suhtautua kuten erilaisiin energialajeihin, joiden välillä on omat muunnoksensa, eli kyse on lopulta samasta asiasta.

            Tuli muuten mieleen ajatuskoe. Jos kaksi erimassaista kappaletta kulkee rinnakkain samansuuntaisesti sekä ulkopuolisen tarkkailijan kannalta vakionopeudella, ja pieneen massaan kohdistetaan jarruttava vastavoima, joka saa kappaleen pysähtymään ulkopuolisen tarkkailijan koordinaatiston origon kohdalla, niin mistä suuren kappaleen liike-energia, joka pienempi kappale havaitsee, oikein kumpuaa – sitä kun ei voi taikoa tyhjästä? Jos siis kaikki ovat omalta osaltaan oikeassa, niin eikö kappaleisiin vaikuttava hitausvoima ole yksi yhteen jonkin kiihtyvyyttä mallintavan koordinaatiston kanssa? Ja tuohon kaiken kattavaan koordinaatistoon vaikuttaa sitten kaikki mahdollinen massa-energia, jolloin mielivaltaisen kappaleen hitaus on jo kosminen ilmiö 🙂

          2. Syksy Räsänen sanoo:

            Kysymäsi tilanne on sama kuin seuraava. Otat kanssasi levossa olevan pallon käteen ja heität sen. Pallo saa liike-energiaa koska kohdistat siihen voimaa. Tämä ei liity yleiseen suhteellisuusteoriaan, eikä siitä tässä sen enempää.

  5. Nova sanoo:

    Ilmeisesti Maan liikettä Auringon ympäri voi ajatella siten, että Aurinko kaareuttaa avaruutta ja Maa liikkuu kyllä suoraviivaisesti, mutta tätä kaarevaa avaruutta pitkin? Tämän ymmärrän siten, että Maa on jo valmiiksi liikkeessä ja seuraa Auringon kaareuttamaa ”rataa”. Kuinka yleisen suhteellisuusteorian mukaan tulisi ajatella haukansulan pudottamista Kuussa? Mikä saa sulan liikkeeseen kohti Kuun pintaa, kun se päästetään irti ja Kuu ei sitä Newtonin gravitaation omaisesti vedä puoleensa?

    1. Syksy Räsänen sanoo:

      Newtonin klassisen mekaniikan mukaan ei ole fysikaalista eroa, onko paikallaan vai liikkuuko vakionopeudella, eli se yhdisti levon ja tasaisen liikkeen. Suppeassa suhteellisuusteoriassa tilanne on sama.

      Yleisen suhteellisuusteorian mukaan ei ole fysikaalista eroa, onko paikallaan, liikkuuko vakionopeudella vai putoaako gravitaatiokentässä. Einstein käytti teoriaa muotoillessaan ajatuskoetta siitä, että on hississä, jonka vaijeri on leikattu. Mistä tietää putoaako vai onko paikallaan painottomassa tilassa? Vastaavasti Galilei teki ajatuskokeen siitä, että on tasaisesti kulkevan laivan hytissä. Mistä tietää, onko paikallaan vai liikkeessä?

      Kun Scott piti haukansulkaa kädessään, se liikkui suoraa reittiä aika-avaruudessa. Reitin suunnan (kohti Kuun keskustaa) määrää se, miten Kuu kaareuttaa aika-avaruutta.

      Samaan tapaan sähkömagnetismissa sähköisesti varattujen hiukkasten välillä ei ole suoria voimia: varattu hiukkanen saa aikaan sähkömagneettisen kentän, joka kohdistaa toisiin hiukkasiin voimia. Yleisessä suhteellisuusteoriassa hiukkanen muuttaa aika-avaruuden kaarevuutta, ja tämä kaarevuus muuttaa toisten hiukkasten liikettä.

  6. Jari Toivanen sanoo:

    Sähkömagneettisia hiukkasia on + ja – merkkisiä, samoin spin voi olla + tai -. Miksi Higgsin kenttä antaa kappaleille (liekö tuo edes oikein sanottu?) vain positiivisia massoja? Onko esitetty teoriaa, joka sallisi kappaleille myös negatiivisen massan?

    1. Syksy Räsänen sanoo:

      Higgsin kenttä antaa alkeishiukkasille massat (paitsi ehkä neutriinoille, niiden massojen alkuperästä ei ole varmuutta). Mutta suurin osa tavallisen aineen hiukkasten massoista (eli energioista) tulee kvarkkien ja gluonien -joista protonit muodostuvat- sidosenergioista. Higgsin kentän antaman massan osuus näkyvän aineen massasta on vain jokunen prosentti.

      Asiaa on tutkittu paljon.

      Yleisessä suhteellisuusteoriassa gravitaation lähteenä ei ole vain massa eikä edes vain energia, vaan muutkin aineen ominaisuudet. Se, vetääkö kappale puoleensa vai hylkiikö riippuu siitä, minkämerkkinen on sen ( energiatiheys + kolme kertaa paine ).

      Teoriat, joissa energiatiheys voi olla negatiivinen ovat usein epöstabiileja (mutta eivät välttämättä – asia on monisyinen). Toisaalta jos paine taasen on tarpeeksi negatiivinen, gravitaatio voi olla hylkivä. Näin on pimeän energian kohdalla: sen energiatiheys on positiivinen mutta paine negatiivinen, joten sen gravitaatio hylkii.

      1. Erkki Kolehmainen sanoo:

        Paine, sillä tavalla kuin minä sen ymmärrän, ei voi olla negatiivinen, koska paine johtuu molekyylien tai atominen törmäyksistä! Niitä joko on tai ei ole! Wittgenstein tuli jossain vaiheessa elämäänsä siihen tulokseen, että kaikki filosofiset ongelmat ovat kielen ongelmia. Ja kun seuraa täällä käytävää keskustelua, niin tulee vääjäämättä mieleen, että niin ovat myös fysiikan ongelmat!

        1. Syksy Räsänen sanoo:

          Paine on yleisempi käsite kuin hiukkasten törmäyksiin liittyvä työntäminen.

          1. Martti V sanoo:

            Tarvitaanko hylkivää substanssia? Eikö riitä, että energiantiheyden ollessa riittävä avaruus kaareutuu riittävästi aiheuttaen näennäisen vetovoiman?

          2. Syksy Räsänen sanoo:

            Hylkiminen ja vetovoima ovat toistensa vastakohtia.

          3. Martti V sanoo:

            Vastaus oli melko trviaali. Asetan väitteen: Massat eivät vedä puoleensa eikä siis hylji. Kaikkea liikettä ohjaa aika-avaruus, jolla on taipumus laajentua/venyä. Massat valuvat lokaaleihin keskittymiin.

          4. Syksy Räsänen sanoo:

            Yleisessä suhteellisuusteoriassa tosiaan massat eivät suoraan vedä toisiaan puoleensa. Ne kaareuttavat aika-avaruutta, joka sitten vaikuttaa kappaleiden liikkeisiin. (Vastaavasti sähkövaraukset eivät suoraan vedä toisiaan puoleensa: ne vaikuttavat sähkökenttään, joka vaikuttaa varattujen kappaleiden liikkeisiin.)

            Tarkemmin aiheesta, ks.

            https://web.archive.org/web/20220925025009/http://www.tiede.fi/blogit/maailmankaikkeutta_etsimassa/kaareuden_kietoutumista

            https://www.ursa.fi/blogi/kosmokseen-kirjoitettua/painon-valittajasta/

            https://www.ursa.fi/blogi/kosmokseen-kirjoitettua/eroon-suuresta-jarjettomyydesta/

  7. Miguel sanoo:

    Tämä ei ehkä liity kuin köykäisesti aiheeseen, mutta kun neutriinot on mainittu. Jos neutriinoilla on massa ”niiden massojen alkuperälle ei ole varmuutta”. Neutronit oskilloivat eli muuttuvat. Meneekö tämä jotenkin hierarkisesti raskaasta kevyeen vai miksi nuo kevyemmät muuttuisi raskaammaksi ja millä energialla? Musta näkymättömät neutriinot on kiinnostavia. Se, mitä et näe on kiinnostavaa.

  8. Cargo sanoo:

    Eikö tuota kappaleiden putoamisen samatahtisuutta voisi perustella ihan maalaisjärjen avulla: Jos laittaa kasan tiiliskiviä päällekäin, niin tyhjiössä ne putoavat samaan tahtiin kuin yksittäinen tiiliskivi. Kun kerta kappaleiden välillä ei ole hylkiviä painovoimia, niin yhdistetyn systeemin kokema putoamiskiihtyvyys on sama kuin sen komponenteilla.

    1. Syksy Räsänen sanoo:

      Tämä ei selitä sitä miksi sulka ja vasara putoavat samalla tavalla, kun niiden koostumus on erilainen.

      1. Cargo sanoo:

        Mikä ylipäätänsä kelpaa selitykseksi? Tässäkin tapauksessa kyse on kaavan E = mc^2 mukaisesti täysin samasta tavasta, johon painovoima vaikuttaa. Jos kuvitellaan tyhjiö, jossa ulkoinen painovoima on kytkettynä pois, ja kootaan haluttu kappale pienistä sähköisesti neutraaleista murusista, niin painovoiman kytkeytyessä päälle, putoaa koko helahoito samaan tahtiin, ad infinitum. Mutta Jumalakaan ei voi meitä auttaa, jos alamme pohtimaan miksi punainen ja sininen pallo tippuvat samaan tahtiin.

        1. Syksy Räsänen sanoo:

          Nyt oletat sen mikä pitäisi osoittaa: että hidas massa ja painava massa ovat sama asia.

          1. Cargo sanoo:

            Kiitoksia vastauksista. Mutta haluaisin kysyä vielä yhden tarkentavan kysymyksen.

            Jos kappaleen A hitausmassa m määrää sen energiasisällön, E = mc^2, niin eikö ko. kappale lakkaa olemasta eikä sillä siten voi olla mitään liike- tai painovoimaenergiaa, jos siitä poistetaan energia E. Jäljelle jää siis pelkkä aineeton varjo. Tämä energia E ei häviä vaan sen avulla muodostetaan jokin toinen aineellinen kappale B, jolla on sama hitausmassa m. Puristetaan sekä kappale A että kappale B kuutioiksi ja muunnetaan niiden kokonaisenergia painovoimakentän määräämäksi energiaksi. Jos nyt soveltaa tuota ”tiiliskiviperiaatetta”, niin eivätkö kappaleet A ja B putoa samaan tahtiin? Ja taas toisaalta, jos ne eivät putoa samalla tavalla, niin millainen metafysikaalinen porsaanreikä sen voisi mahdollistaa? Mutta joo, en jatka enää tästä aiheesta, sillä alkaa tuntumaa hieman siltä, että fyysikot problematisoivat eriväristen pallojen putoamista – hieman samalla tavalla kuin heidän ylenkatsomansa filosofit.

            P.S. Tämän blogimerkinnän aiheesta tulikin mieleen aasinsilta, jolla taas tölväistä filosofeja. Vapaapudotus jossa ei huomioida ilmanvastusta on yhtä realistinen malli kuin Minkowskin aika-avaruus jossa ei huomioida epämääräisyysperiaatetta; ensimmäinen johtaa äärettömään nopeuteen ja jälkimmäinen eternalistiseen aikakäsitykseen.

          2. Syksy Räsänen sanoo:

            Yhtälö E = m c^2 pätee vapaalle hiukkaselle joka ei liiku. Massattomat hiukkaset liikkuvat aina valonnopeudella, joten yhtälö ei pöde niille. Yleinen yhtälö, joka pätee kaikille vapaille hiukkasille on E^2 = m^2 c^4 + p^2 c^2, missä p on hiukkasen liikemäärä.

            Ei tästä enempää.

Vastaa

Sähköpostiosoitettasi ei julkaista. Pakolliset kentät on merkitty *

Viestinviejä naulavuoteella

16.1.2023 klo 13.23, kirjoittaja
Kategoriat: Kosmokseen kirjoitettua , Kosmologia

Puhun tiistaina 14.2. kello 18 Kirkkonummen Komeetan tilaisuudessa Kirkkonummen pääkirjaston (Kirkkotie 1) Mörne-salissa otsikolla ”Valo maailmankaikkeudessa: viestinviejä naulavuoteella?”. Aiheena on valo kosmologiassa ja sen kulku halki kaarevan aika-avaruuden. Selitän myös jonkin verran omasta tutkimuksestani, jota käsittelin tässä merkinnässä.

Päivitys (13/02/22): Sairastumisen takia puhe on siirretty syyskauteen. Laitan tiedon uudesta ajankohdasta myöhemmin.

3 kommenttia “Viestinviejä naulavuoteella”

  1. Tapaamisiin 14.2 Itis

  2. Nolomies99 sanoo:

    ei liity suoranaisesti mihinkään, mutta; nöyräpyyntö

    voidaanko casimir ilmiötä pitää ”negatiivisena energiana” koska levyjen ulkopuolinen avaruus ei ole ”tyhjä”
    Tyhjä ainoastaan klassisen mekaniikan näkemyksenä.

    kiitos!

    1. Syksy Räsänen sanoo:

      Casimir-ilmiössä on kyse siitä, että sähköisesti varatut hiukkaset vetävät toisiaan puoleensa. Se on kvanttikorjaus klassiseen sähkömagneettiseen voimaan. Siihen liittyy negatiivinen energia, kuten sähkömagneettiseen vuorovaikutukseen yleensäkin.

      Tieteellinen artikkeli aiheesta alla; sen johdanto- ja johtopäätös-osioita voi ymmärtää ilman erikoistietoja.

      https://arxiv.org/abs/hep-th/0503158

      Ei tästä sen enempää, kun ei liity merkinnän aiheeseen.

Vastaa

Sähköpostiosoitettasi ei julkaista. Pakolliset kentät on merkitty *


Tuulien kääntymistä

22.12.2022 klo 19.24, kirjoittaja
Kategoriat: Kosmokseen kirjoitettua

Hiukkasfysiikan Standardimalli on yksi ihmiskunnan suuria saavutuksia. Se kuvaa kaikkia tunnettuja hiukkasia ja vuorovaikutuksia gravitaatiota lukuun ottamatta. Standardimalli on ennustanut yli neljän vuosikymmenen ajan oikein kaikkien hiukkaskiihdyttimissä tehtyjen kokeiden tulokset, jotkut miljardisosan tarkkuudella.

Kokeellisen menestyksen taustalla on hienostunut teoreettinen rakenne nimeltä kvanttikenttäteoria, joka yhdistää kvanttimekaniikan ja suppean suhteellisuusteorian. Ensimmäinen Standardimallin osa, sähkömagneettista vuorovaikutusta kuvaava kvanttielektrodynamiikka, löydettiin 1948. Se selitti ja ennusti tarkasti vetyatomin rakenteen ja muita ilmiöitä. Vuonna 1954 kuitenkin Aleksei Abrikosov, Isaak Khalatnikov ja Lev Landau osoittivat, että sähkömagneettisen vuorovaikutuksen voimakkuus kasvaa energian myötä ja muuttuu lopulta äärettömäksi, mikä on matemaattisesti ristiriitaista.

Monet päättelivät tästä, että kvanttikenttäteoriassa on jotain perustanlaatuista vikaa ja hiukkasfysiikassa pitää suunnata muille reiteille. Kaksi tapahtumaa vaikutti merkittävästi siihen, että kvanttikenttäteoria palasi suosioon ja Standardimallista tuli standardi.

Yksi läpimurto tuli vuonna 1973 Standardimallin toisesta osasta. Silloin osoitettiin, että värivuorovaikutusta kuvaavassa kvanttikromodynamiikassa voimakkuus pienenee energian myötä. Tämä osoitti, että kaikilla kvanttikenttäteorioilla ei ole Landaun ja kumpp. löytämää ongelmaa.

Toinen mullistava tulos oli saatu kahta vuotta aiemmin, kun jatko-opiskelija Gerardus ‘t Hooft ja hänen väitöskirjaohjaajansa Martinus J. G. Veltman osoittivat, että Standardimallissa hiukkasilla on vain äärellinen määrä tapoja vuorovaikuttaa keskenään.

Kvanttikenttäteorioissa kvanttifysiikan vuorovaikutusten kokonaisuus on hyvin monimutkainen. Siksi kvanttikenttäteorioita rakennetaan lähtemällä teoriasta, jossa ei ole mukana kvanttifysiikkaa, ja kvanttiefektejä otetaan mukaan pala kerrallaan. Kvanttiefektit voivat joka askeleella tuoda teoriaan uusia vuorovaikutuksia, joita ei alun perin ollut mukana. (Murray Gell-Man nimesi tämän totalitaristiseksi periaatteeksi: kaikki mikä ei ole kiellettyä on pakollista.)

Asiaa voi havainnollistaa vertaamalla Isaac Newtonin gravitaatioteoriaan, missä kappaleet vetävät toisiaan puoleensa voimalla, joka on kääntäen verrannollinen niiden etäisyyden neliöön. Jos kyseessä olisi kvanttikenttäteoria, kvanttiefektit toisivat gravitaatiovoimaan mukaan osia, jotka heikkenevät nopeammin etäisyyden kasvaessa: kuten etäisyyden kolmas potenssi, neljäs potenssi, ja niin edespäin.

Suurimmassa osassa kvanttikenttäteorioita on niissäkin äärettömän monta erilaista vuorovaikutusta, eikä teoriasta voi laskea, miten voimakkaita ne ovat. Niinpä oikeastaan mitään ei voi ennustaa, koska on äärettömän monta numeroa, joita sovittaa havaintoihin.

Joidenkin poikkeuksellisten kvanttikenttäteorioiden rakenne kuitenkin sallii vain äärellisen (ja yleensä pienen) määrän erilaisia vuorovaikutuksia. ’t Hooft ja Veltman osoittivat, että Standardimalli on tällainen teoria. Tämä oli mullistava tulos: se osoitti, että Standardimalli on vakaalla pohjalla ja sen ennustuksiin voi luottaa. Standardimallista tuli hiukkasfysiikan vuorenhuippu, ja ’t Hooftille ja Veltmanille myönnettiin Nobelin palkinto vuonna 1999.

Tuloksen merkitystä kuvaa se, että se palkittiin Nobelilla, vaikka ’t Hooft ja Veltman eivät ennustaneet eivätkä löytäneet mitään, vaikka palkinnon lehdistötiedotteessa Standardimalliin liittyviä kokeita korostetaankin. Yleensä fysiikan Nobeleita annetaan vain kokeellisesti varmistetuista löydöistä.

Vuosikymmenien varrella suhtautuminen ’t Hooftin ja Veltmanin tulokseen on kuitenkin muuttunut. Eräs hiukkasfysiikan kärkihahmona pidetty teoreetikko on peräti sanonut, että sillä ei ole mitään merkitystä.

Syynä on se, että Standardimalliin ja muihin hiukkasfysiikan teorioihin on ruvettu suhtautumaan vain approksimaatioina, joiden pätevyysalue on rajallinen. Tällöin ei haittaa, vaikka teoriasta ei voi tehdä tarkkoja ennustuksia tai se ei ole ristiriidaton, kunhan siitä voi ennustaa jotain, vaikka vähemmän perustellusti. Tämä lähestymistapa tunnetaan nimellä efektiivinen kenttäteoria, missä ensimmäinen sana viittaa siihen, että teoria toimii vain rajatulla alueella.

Palataan vertaukseen, missä kvanttiefektit lisäisivät Newtonin gravitaatiovoimaan osia, jotka riippuvat eri tavalla etäisyydestä. Jos teoria käyttäytyy kuten Standardimalli, erilaisia osia on vain pieni määrä. Jos se käyttäytyy kuten efektiivinen kenttäteoria, voiman osia on äärettömästi, mutta ne joiden voimakkuus laskee nopeasti etäisyyden kasvaessa voi sivuuttaa kun kappaleet ovat tarpeeksi kaukana toisistaan. Teoria siis pätee isoilla etäisyyksillä, kunhan nopeasti heikkenevät osat ovat tarpeeksi pieniä eikä lasketa liian tarkasti.

Samalla kun ’t Hooftin ja Veltmanin tuloksen arvostus on laskenut, heidän sen todistamisessa käyttämänsä renormalisaationa tunnetun menetelmän merkitys on noussut yhdessä efektiivisen kenttäteorian myötä. Renormalisaatiossa lasketaan miten pienen mittakaavan ilmiöt vaikuttavat isomman mittakaavan tapahtumiin. Renormalisaatiota käytettiin hiukkasfysiikassa jo ennen ’t Hooftin ja Veltmanin työtä, ja nykyään se on osana efektiivistä kenttäteoriaa keskeinen työkalu fysiikan eri aloilla, kuten kiinteän olomuodon fysiikassa (vaikkapa puolijohteiden ymmärtämisessä) tai sen tutkimisessa, millaisia rakenteita galaksit muodostavat.

Toisaalta Standardimallin suhteen tuuli on taas kääntymässä. Vastoin odotuksia hiukkaskiihdytin LHC ei olekaan löytänyt uutta fysiikkaa, vaan on sen sijaan jatkanut Standardimallin ennusteiden varmentamista. Tämä on saanut jotkut tutkijat ottamaan Standardimallin entistä vakavammin. Teoriat joissa hiukkasilla on vain äärellinen määrä tapoja vuorovaikuttaa ovat hyvin poikkeuksellisia. Onko sittenkään sattumaa, että Standardimalli, joka kuvaa havaintoja odotettua paljon paremmin, on tällainen teoria? Asiasta ei ole selvyyttä, ja voi olla että Standardimallin ymmärtämisessä on edessä vielä uusi läpimurto.

30 kommenttia “Tuulien kääntymistä”

  1. Martti V sanoo:

    ”Joidenkin poikkeuksellisten kvanttikenttäteorioiden rakenne kuitenkin sallii vain äärellisen (ja yleensä pienen) määrän erilaisia vuorovaikutuksia”. Millaisista teorioista tässä on kyse?

    1. Syksy Räsänen sanoo:

      Tällaisia teorioita sanotaan renormalisoituviksi teorioiksi. Niiden rakenteen selittäminen vaatisi pidemmän tekstin, mutta sanon karkeasti ja epäselvästiu, että tyypillinen renormalisoituva teoria on sellainen, jossa 1) vuorovaikutuksiin liittyy tietynlainen symmetria (mittasymmetria), 2) kaikkien vuorovaikutusten voimakkuutta mittaavat varaukset (kuten sähkövaraus) ovat dimensiottomia yksikköjärjestelmässä, missä valonnopeuson yksi ja redusoitu Planckin vakio on yksi, ja 3) hiukkassisältö on sopiva.

  2. Eusa sanoo:

    Symmetriat ovat mielenkiintoisia. Supersymmetria perustuu oletuksiin hiukkasperheestä. Jospa supersymmetria onkin levittäytyneemmissä virityksissä?

    Tuntuisi, ettei hyviä symmetriaideoita välttämättä tarvitsisi heittää romukoppaan vaan niille voisi löytyä perusteluja ja todennettavaa näkökulman vaihdolla. Liekö tosissaan vielä yritettykään?

    1. Syksy Räsänen sanoo:

      Supersymmetria ei perustu ”oletuksiin hiukkasperheestä”. Standardimallin perherakenteella ei ols supersymmetrian kanssa mitään tekemistä.

      Supersymmetria perustuu oletukseen, että suppean suhteellisuusteorian aika-avaruutta kuvaavat symmetriat ja hiukkasfysiikan hiukkasten vuorovaikutuksiin liittyvät symmetriat ovat osa samaa kokonaisuutta. Tästä oletuksesta seuraa, että hiukkasia pitää olla havaittuja enemmän.

  3. M. Hyvönen sanoo:

    Hei,

    Asiallinen, asiantunteva ja mielenkiintoinen artikkeli!

    Ursassa tiedätte varsin hyvin, että Räsänen on saanut syytteet kahdesta törkeästä rikoksesta. Käräjäoikeuden vapauttava päätös ei ole vielä lainvoimainen.

    Yhdistyksenä Ursan tulisi täysin pidättäytyä kaikesta yhteistyöstä Räsäsen kanssa rikosprosessin ollessa vireillä. Tämä on nykyisin täysin vakiintunut käytäntö. Tämä osoittaisi myös, että Ursa ei hyväksy mitään rikollista toimintaa. Epäillyt syyksiluettavat tekomuodot ovat törkeitä ja niitä on kaksi kappaletta.a

    1. Syksy Räsänen sanoo:

      Vastaan omalta osaltani.

      Oikeus hylkäsi syytteet kunnianloukkauksesta ja yksityiselämää loukkavasta tiedon levittämisestä. Päätös on lainvoimainen. Käräjäoikeus totesi, että emme olleet rikkoneet lakia, ja että käsittelemäämme aihetta voidaan ”pitää sellaisena yleisen edun mukaisena asiana, josta tulee voida avoimesti keskustella”. Oikeus lisäksi totesi, että ”asialla on oletettavasti ollut merkitystä koko pienelle tiedeyhteisölle ja se on ollut yhteiskunnallisesti ja myös maailmanlaajuisesti merkityksellinen ja laajaa keskustelua herättänyt asia, mikä korostaa sananvapauden suojaa”.

      Tarkemmin, ks. https://www.ursa.fi/blogi/kosmokseen-kirjoitettua/myos-me-kierros-4-we-too-round-4/

      Väitteesi ”täysin vakiintuneesta käytännöstä” ei pidä paikkaansa. Esimerkiksi Johanna Vehkoon viisi vuotta kestäneen kunnianloukkausjutun aikana journalistiset tahot lisäsivät yhteistyötä hänen kanssaan ja antoivat hänelle tunnustuksia.

  4. Erkki Kolehmainen sanoo:

    Pelkään pahasti, että Standardimallin ohjaava voima on niin suuri, että sen perusteella tehdyt koejärjestelyt (LHC) antavat aina tuloksen, joka vahvistaa Standardimallia! Eli suljetussa luupissa ollaan eikä tuulet miksikään käänny ennen kuin päästään mallista eroon. Nyt tarvitaan ajattelussa kvanttihyppäyksen kaltaista laadullista muutosta?

    1. Syksy Räsänen sanoo:

      Standardimallin tuonpuoleista fysiikkaa etsitään kymmenillä erilaisilla koejärjestelyillä LHC:ssä.

  5. Erkki Kolehmainen sanoo:

    Kiitokset Syksylle siitä, että hän uskalsi mainita kolmen venäläisen fyysikon nimet positiivisessa sävyssä. Sellaista ei viime aikoina ole juurikaan tapahtunut, koska Venäjästä ja venäläisyydestä on tehty kirosana!

    1. Syksy Räsänen sanoo:

      Khalatnikov syntyi Ukrainassa. Landau syntyi nykyisen Azerbaidžanin alueella ja työskenteli Ukrainassa ja Venäjällä. En tiedä millainen kansallinen identiteetti heillä oli, itse puhuisin neuvostofyysikoista.

  6. Voiko standardimallin kaikkia parametreja (joita muistaakseni on parisenkymmentä) varioida ilman että renormalisoituvuus häviää? Voiko hiukkasperheitä olla jokin muu määrä kuin kolme, ilman että renormalisoituvuus häviää?

    1. Syksy Räsänen sanoo:

      Renormalisoituvuus säilyy riippumatta parametrien arvoista ja perheiden määrästä.

      1. Minusta standardimalli on vähän kuin vuoristomaisema, joka katselemme kivikautisesta alppikylästä. Vuorten yleispiirteet vihjaavat taustalla olevasta geofysiikasta (vuorenpoimutus, laattatektoniikka, ruuhilaaksot, jääkaudet, jne.), mutta yksittäisten vuorten paikat ovat satunnaisia. Haasteena on katsoa standardimallia niin että taustalla oleva fysiikka paljastuisi ilman että satunnaiset yksityiskohdat varastavat huomion; helpommin sanottu kuin tehty. Geofysiikan tapauksessa pelkkä vuoriston katselu ei riittänytkään, vaan lisäksi piti kiinnittää huomio rinteiltä löytyviin merieläinten fossiileihin sekä maailmankartan muotoihin, jotta päästiin avainidean eli mannerliikuntojen jäljille.

  7. Erkki Kolehmainen sanoo:

    Vastaus ongelmaan on tämä: Mordehai Milgrom ja MoND (Modified Newtonian Dynamics)

    1. Syksy Räsänen sanoo:

      MOND on yritys selittää osaa niistä havainnoista, joita pimeä aine selittää, ilman pimeää ainetta. Se ei ole kokonainen teoria, mutta on kehitetty teorioita joiden rajatapauksena olisi MOND. Yksikään tällainen teoria ei ole pystynyt selittämään kaikkia samoja havaintoja kuin pimeä aine, saati onnistuneesti ennustamaan uusia kuten pimeä aine.

      Tarkemmin, ks.

      https://www.ursa.fi/blogi/kosmokseen-kirjoitettua/luodin-jaljet/

  8. Harri Pohja sanoo:

    Kiitos taas mielenkiintoisesta katsauksesta! Onko Standardimallista olemassa jokin ”virallinen versio”, jota päivitetään tiedeyhteisön konsensuksen mukaan? Esimerkiksi omien opintojeni aikaan neutriinoilla ei ajateltu olevan nollasta poikkeavaa lepomassaa, 90-luvulla ilmeisesti neutriinojen kätisyyden vaihtuminen alkoi edellyttää lepomassaa, pari–kolme vuotta sitten mainitsit blogissasi neutriinojen massaksi 5–10 keV, ja uusimmissa artikkeleissa näyttää massan arvion tarkentuneen johonkin 0,1 eV paikkeille (jos olen oikein ymmärtänyt). Ovatko nämä arviot massoista aikojen kuluessa vain jonkinlaisia välituloksia, vai hyväksytäänkö ne standardimalliin sitä mukaa kuin arviot muuttuvat?

    1. Syksy Räsänen sanoo:

      Tämä onkin hyvä kysymys. Standardimallista on periaatteessa vain yksi versio, jossa tosiaan ei ole neutriinoiden massoja. Jotkut tosin kutsuvat Standardimalliksi myös teoriaa, johon on lisätty neutriinoiden massat, koska ei ollut hyvää syytä jättää niitä alun perin pois.

      Neutriinoiden massojahan ei ole havaittu kiihdyttimissä, vaan toisenlaisissa kokeissa. Massojen tarkkoja arvoja ei tiedetä, vain yläraja (joka on tosiaan noin 0.1 eV) ja neutriinoiden emassojen erot.

      Tarkemmin, ks.

      https://www.ursa.fi/blogi/kosmokseen-kirjoitettua/sekoittumista/

      http://www.tiede.fi/blogit/maailmankaikkeutta_etsimassa/pikkupuolueettomien_epamaaraisyytta

      http://www.tiede.fi/blogit/maailmankaikkeutta_etsimassa/pikkupuolueettomien_taustoja

      To 5-10 keV on massa-arvio kevyimmälle nk. oikeakätiselle neutriinolle, jos pimeä aine koostuu siitä. Oikeakätisten neutriinoiden olemassaolosta ei ole varmuutta.

      Tarkemmin, ks.

      https://www.ursa.fi/blogi/kosmokseen-kirjoitettua/huippujen-laskeminen/

      https://www.ursa.fi/blogi/kosmokseen-kirjoitettua/vasemmalta-oikealle/

      https://www.ursa.fi/blogi/kosmokseen-kirjoitettua/kauneusvirheen-korjaaminen/

      1. Erkki Kolehmainen sanoo:

        Neutriinoista puheen ollen Bruno Pontecorvon nimi on mainittava. Hän oli Enrico Fermin oppilas, joka loikkasi Helsingin kautta Neuvostoliittoon v. 1950 ja teki pääosan tieteellisestä urastaan siellä. Hänen ideansa on mm. neutriinoiden oskillaatio eli muuttuminen toisikseen!

      2. Harri Pohja sanoo:

        Kiitos pikaisesta ja kattavasta vastauksesta! Noissa aiemmissa kirjoituksissasi mainitset nuMSM:n muutaman kerran, ja että se ”on hyvin motivoitu, yksinkertainen ja vieläpä kokeellisesti lähiaikoina varmennettavissa tai poissuljettavissa” (lainaus 2016 blogistasi). Mikähän sen osalta on nykyään tilanne, onko esimerkiksi kokeellisesti onnistuttu varmentamaan nuMSM:ää suuntaan tai toiseen?

        P.S. Nuo linkit Tiede-lehden blogitesteihin vievät vain Helsingin Sanomien Tiede-osastolle, enkä onnistunut sitä kautta mitenkään pääsemään käsiksi noihin teksteihisi. Arkiston kautta ehkä saattaisi päästä, mutta se taitaa olla vain Tiede-lehden tilaajille?

        1. Syksy Räsänen sanoo:

          nuMSM:n tilanne ei ole oleellisesti muuttunut. Kolme pientä askelta on otettu sitten vuoden 2016.

          1) Havainnot ovat rajoittaneet oikeakätisten neutriinoiden massaa, jos ne ovat pimeää ainetta ja niiden massa on 1-10 keVin tienoilla. Massaikkuna ei kuitenkaan ole sulkeutunut.

          2) Tarkemmat laskut baryogeneesistä osoittavat, että raskaampien oikeakätisten neutriinoiden (niiden jotka eivät ole pimeää ainetta) massaikkuna onkin luultua paljon isompi.

          3) Jotkut röntgensädehavainnot saattavat olla syntyneet oikeakätisten pimeän aineen neutriinoiden hajoamisesta.

          Tosiaan, Tiede-lehden blogithan on lakkautettu. Tekstit löytyvät laittamalla linkin Wayback Machineen. Yritän jatkossa muistaa pistää Wayback Machine -linkit noiden vanhentuneiden linkkien sijaan.

          https://archive.org/web

      3. Eusa sanoo:

        Neutriinojen massattomuus ja itsensä antihiukkaisuus, oikeakätisenä antihiukkasensa, ei liene vieläkään todennettu mahdottomaksi, järkeväksi epäillä sen sallivaa mekanismia massaoskilloinnin sijaan? Tässäkin levittäytyneen funktion sumea logiikka voisi olla vahva työkalu.

        1. Syksy Räsänen sanoo:

          Jos neutriinot ovat omia antihiukkasiaan, se ei tarkoita, että ne olisivat massattomia. Itse asiassa kun neutriinoilla on massat, on luontevinta, että ne ovat silloin omia antihiukkasiaan. (Mutta se ei ole välttämätöntä.)

          1. Eusa sanoo:

            Siis yhdistelmä massattomuus-majoranaisuus-fermionisuus on mahdollinen, vaikka massallisuus näyttääkin nyt vakuuttavammalta selitysmallilta.

          2. Syksy Räsänen sanoo:

            Neutriinot ovat fermioneja. Se, että ne olisivat massattomia ja Majorana-fermioneja (eli omia antihiukkasiaan) ei mitenkään selittäisi havaintoja neutriinoiden oskillaatioista, sitä varten niillä pitää olla massat.

            Tämä riittäköön tästä.

          3. Martti V sanoo:

            Eikös hiukkasella ja sen anti-parilla ole aina sama massa? mainitsit, että neutriinojen (vas. kät.) massan yläraja olisi 0.1eV. -> eikö se ole silloin myös oikeakätisen? Neturiinoja on kolmessa generaatiossa. Kuuluko pimeänaineen kandidatti niistä tiettyyn?

          4. Syksy Räsänen sanoo:

            Kyllä, mutta vasenkätiset ja oikeakätiset neutriinot eivät ole toistensa antihiukkasia.

            Mainitsemassani nuMSM-mallissa (ks. kommentit yllä) kevyin oikeakätinen neutriino on pimeän aineen hiukkanen. Kaksi muuta voivat olla vastuussa aineen ja antiaineen epäsuhdasta, ks. https://www.ursa.fi/blogi/kosmokseen-kirjoitettua/vasemmalta-oikealle/

            Kyse tosin ei ole aivan perheistä sikäli, että ne neutriinot, joilla on tarkat massat, ovat sekoitus eri perheiden neutriinoja. Tavallisten vasenkätisten neutriinoiden kohdalta yritin selittää asiassa täällä: https://web.archive.org/web/20220820021821/http://www.tiede.fi/blogit/maailmankaikkeutta_etsimassa/pikkupuolueettomien_epamaaraisyytta

  9. Grigori Melehov sanoo:

    onko esitetty teoriaa, jossa aika on kolmiulotteinen?

    1. Syksy Räsänen sanoo:

      Luultavasti. Ainakin kahta aikaulottuvuutta on pohdittu. Menee sen verta kauas merkinnän aiheesta, että ei siitä enempää.

  10. Martti V sanoo:

    ”Kun tavalliset neutriinot muuttuvat oikeakätisiksi neutriinoiksi, leptoniluku siis muuttuu. ” tästä sain käsityksen, että vasen ja oikeakätinen ovat toisetensa antihiukkasia. Neutriino on oma antihiukkasensa, sillä varaus on sama nolla. Miten spin suhtautuu?

    1. Syksy Räsänen sanoo:

      Kun merkintä ei koske neutriinoja, ei tästä sen enempää.

Vastaa

Sähköpostiosoitettasi ei julkaista. Pakolliset kentät on merkitty *

Naulavuoteella kävelemistä

14.12.2022 klo 21.40, kirjoittaja
Kategoriat: Kosmokseen kirjoitettua , Kosmologia

Vuoden lopulla Helsingin yliopiston fysiikan osasto pyytää tutkijoita kirjoittamaan ymmärrettävän tiivistelmän jostakin vuoden aikana julkaistusta tuloksesta, joita se sitten nostaa sivuilleen. Viime vuonna poimin esimerkiksi työmme mustien aukkojen parissa (löytyy otsikon ”Quantum kicks have a big effect on abundance of Eros-mass black holes” alta), josta olen kirjoittanut täällä blogissakin. Tänä vuonna valitsin Sofie Marie Koksbangin ja minun artikkelin siitä, miten pimeän aineen hiukkasluonne vaikuttaa valon kulkuun.

Suurin osa päätelmistämme maailmankaikkeudesta perustuu valoon. Laskettaessa ennusteita sille, miltä maailmankaikkeus näyttää, yleensä oletetaan, että valo kulkee suorinta reittiä aika-avaruudessa. Jotta tämä pitäisi paikkansa, valon aallonpituuden pitää olla paljon pienempi kuin aika-avaruuden kaarevuuteen liittyvät etäisyydet, muuten valo poikkeaa suoralta polulta.

Aika-avaruutta voi ajatella maastona, jonka korkeus kuvaa aika-avaruuden kaarevuutta. Valon aallonpituutta voi verrata kävelijän askelpituuteen. Jos askel on paljon pienempi kuin etäisyys, millä maasto kaartuu, niin matkaaja kulkee kuin tasaisella maalla, vaikka hiljakseen menisikin ylös tai alas, ja kävely sujuu samaan tapaan kaikkialla. Jos maasto sen sijaan kaartuu askelkokoon verrattavassa tai pienemmässä mittakaavassa, sen muoto vaikuttaa siihen miten siinä liikkuu.

Valon tapauksessa asiaa voi katsoa myös energian kautta. Mitä pienempi valohiukkasen eli fotonin aallonpituus on, sitä korkeampi sen energia on. Kun fotonin energia on tarpeeksi iso, aika-avaruuden paikalliset vaihtelut eivät vaikuta siihen, vaan se pyyhältää niiden läpi muuttumatta.

Aika-avaruuden kaarevuus riippuu aineen tiheydestä. Mitä tiukemmin massa on pakkautunut, sitä isompi on kaarevuus. Tähtitieteellisten kappaleiden kuten tähtien ja galaksien aiheuttama kaarevuus on mitättömän pieni verrattuna kosmologiassa havaittavan valon energiaan.

Mutta valtaosa maailmankaikkeuden aineesta on (luultavasti) pimeää ainetta. Suurimmassa osassa pimeän aineen malleista se koostuu yksittäisistä hiukkasista. Vaikka hiukkasen massa on paljon pienempi kuin tähden, se on pakkautunut hyvin pieneen tilaan, joten tiheys ja siksi myös kaarevuus voi olla iso.

Vaikuttaako tämä kaarevuus valon kulkuun? Selvittääksemme asiaa Sofie ja minä kehitimme uudenlaisen tavan kuvata valon matkaamista, jossa kaarevuus otetaan huomioon.

Osoittautui, että kun fotoni kulkee pimeän aineen hiukkasen läpi, pimeän aineen hiukkasen aiheuttama kaarevuus antaa fotonille massan, joka on sitä isompi mitä suurempi kaarevuus on. Tämä muistuttaa sitä, miten Higgsin kenttä antaa massan hiukkasille kun ne matkaavat sen läpi. Higgsin kentän tapauksessa tosin on kyse hiukkasfysiikan vuorovaikutuksista, tässä gravitaatiosta. Toisekseen Higgsin kenttä on kaikkialla, kun taas pimeän aineen hiukkasia on harvassa.

Pimeän aineen hiukkasten tyypillinen etäisyys toisistaan on noin metri tai pidempi, hiukkasten massasta riippuen. Käsittelyssämme yhden hiukkasen massa on jakautunut alueelle, joka on hiukkasfysiikan suuruusluokkaa. Aika-avaruuden kaarevuuden maasto koostuu siis toisistaan kaukana olevista kapeista huipuista.

Jos valon energia on iso eli aallonpituus pieni, huiput eivät vaikuta sen kulkuun. Jos valon energia on pienempi kuin kaarevuuden antamaan massaan liittyvä energia, valo ei voi kulkea pimeän aineen hiukkasten läpi, koska sillä ei ole tarpeeksi energiaa jotta pääsisi pimeän aineen sisälle. Samasta syystä arkisessa ympäristössämme näkyvä valo ei pääse sisälle metalleihin. Matkatessaan niissä valo saisi massan, joka on isompi kuin fotonin energia. Jos valon energia tarpeeksi korkea, kuten röntgensäteillä, se kulkee metallienkin läpi.

Huomasimme, että tärkeää on myös se, miten nopeasti kaarevuus muuttuu. Koska hiukkaset ovat pieniä, niiden läpi mentäessä kaarevuus vaihtuu nopeasti. Saimme tulokseksi, että jos pimeän aineen hiukkasen massa on yli 10% protonin massasta, kaarevuuden nopea vaihtuminen muuttaa täysin sen, miltä kosminen mikroaaltotausta näyttää: naulojen jäljet näkyvät. Koska vaikutus riippuu fotonin energiasta, lyhemmän aallonpituuden valo ei sen sijaan juuri muutu: se kävelee ongelmitta naulatyynyllä.

Ajattelimme, että tämä saattaisi selittää kosmologian tämän hetken isoimman ristiriidan havaintojen ja teorian välillä. Kun maailmankaikkeuden laajenemisnopeuden määrittää kosmisesta mikroaaltotaustasta ja supernovista, saa eri tuloksen. Näihin kahteen erilaiseen havaintoon pohjaavassa tarkastelussa on paljon eroja, mutta me kiinnitimme huomiota siihen, että ne pohjaavat erilaiseen valoon. Supernovien valo on näkyvällä ja infrapuna-alueella, eli sen energia on isompi kuin mikroaaltotaustan fotonien.

Löytämämme vaikutus menee kuitenkin havaintojen kannalta väärään suuntaan. Vaikka pimeän aineen massa olisi sopiva, niin että se vaikuttaisi mikroaaltoihin mutta ei infrapunavaloon, tämä vain pahentaisi ristiriitaa. Niinpä käteen jää vain raja sille, miten raskasta pimeä aine voi olla, jotta sen vaikutusta ei näkyisi mikroaaltotaustassa. Raja on suhteellisen tiukka, ja sulkisi pois ison määrän pimeän aineen malleja.

Päättelymme ei kuitenkaan ole aukotonta. Otimme huomioon aika-avaruuden kaarevuuden, mutta jätimme pois muita valon kulkuun vaikuttavia tekijöitä, jotka voivat olla merkittäviä. Käsittelymme siitä, miten massa jakautuu hiukkasten sisällä oli myös hyvin yksinkertainen. Pitäisi tutkia tarkemmin, miten hiukkasten kvanttimekaaninen todennäköisyysjakauma leviää kun ne kelluvat avaruudessa vuorovaikuttaen hyvin heikosti ympäristönsä kanssa.

Voi olla, että johtopäätöksemme vielä muuttuvat, mutta minusta tämä on hauska tapa mahdollisesti saada tietoa pimeän aineen hiukkasluonteesta valon ja gravitaation kautta, ja odotan että pääsen jatkamaan sen parissa vuonna 2023.

18 kommenttia “Naulavuoteella kävelemistä”

  1. Jernau Gurgeh sanoo:

    Mukava lukea omasta tutkimuksestasi, erittäin kiinnostavan oloinen aihe.

  2. Koditon sanoo:

    Fotoni onkin omituinen olio. Lukion fysiikan kirja ei paljasta siitä juuri mitään. Sillä on aallonpituus, taajuus, nopeus jne. Koosta ei mainita mitään, joka olisi se kailkkein kiinnostavin tieto. Onko fotonin koko sama asia kuin sen aallonpituus? Sanot, että hiukkasen koko on pieni. Tarkoitatko, että fotonin koko on pieni. Laserin valoa voi himmentää suodattimilla tasolle, jossa säteen energia on pienempi kuin saman aallonpituuden omaavan fotonin energia. Onko laserin säde nyt yksittäisiä erillisiä fotoneita? Kaksoisrakokokeessahan näin tehdään.
    Mysteeriksi on jäänyt fotonin koko täällä päässä.

    1. Syksy Räsänen sanoo:

      Alkeishiukkasten koko onkin tosiaan monimutkainen kysymys. Kirjoitan merkinnässä vain pimeän aineen hiukkasten koosta.

      Tässä yhteydessä koko viittaa aaltofunktion leveyteen. Hiukkasilla ei ole määrättyä paikkaan, ainoastaan todennäköisyysjakauma paikalle. Niinpä niiden massallakin on todennäköisyysjakauma. Kuten lopussa kirjoitan, ei itse asiassa ole aivan selvää, miten pimeän aineen hiukkasten aaltofunktio kehittyy.

      Fotoneilla on aaltofunktio samalla tavalla.

      Lasersäde koostuu tosiaan suuresta määrästä yksittäisiä fotoneita. Ei kai lasersäteen energia voi laskea yksittäisen fotonin energian alle?

      1. Koditon sanoo:

        Ei taidakaan mennä niin kuin ajattelin.

  3. Lentotaidoton sanoo:

    Räsänen: Niinpä käteen jää vain raja sille, miten raskasta pimeä aine voi olla, jotta sen vaikutusta ei näkyisi mikroaaltotaustassa. Raja on suhteellisen tiukka, ja sulkisi pois ison määrän pimeän aineen malleja.

    Mielenkiintoinen yksi (uusi?) tutkimusnäkökulma. Eli mikä se raja teidän mukaanne olisi? Ja mitä malleja se siten sulkisi pois? Onko tällaisia tutkimuksia muita olemassa, vai oletteko pioneereja?

    1. Syksy Räsänen sanoo:

      Raja on tekstissä mainittu 10% protonin massasta. Pimeä aine ei siis voi olla tuota raskaampaa. Pimeän aineen malleja on satoja erilaisia, suurin osa tämän rajan huonommalla puolella. Tämä on ensimmäinen tutkimus aiheesta. Tulos ei kuitenkaan ole vielä vakaalla pohjalla, approksimaatioidemme pätevyyttä pitää tarkastella huolella.

      1. Lentotaidoton sanoo:

        Eli jos protoni on pyöreästi 1 GeV, niin tuo raja olisi suuruusluokkaa alle Myonin (105 MeV). Tämän mukaan esim WIMPit putoaisivat pois laskuista?

        1. Syksy Räsänen sanoo:

          Joo, raja kosmisesta mikroaaltotaustasta on noin 100 MeViä. Sanaa WIMP käytetään nykyään eri merkityksissä, jotkut käyttävät myös tuota kevyemmille hiukkasille.

          Massaraja kasvaa kuten aallonpituus potenssiin 2/3, eli 21 cm säteilyn (aallonpituus sata kertaa isompi kuin mikroaaltojen) havainnoilla saisi rajan jonnekin MeVin tienoille.

          Arvio massarajalle on karkea: tuolla massalla vaikutus kosmiseen mikroaaltotaustaan olisi luokkaa 100%. Mikroaaltotausta on kuitenkin hyvin tarkkaan mitattu, joten paljon pienempiäkin häiriöitä voisi nähdä, eli tarkemmalla analyysillä massarajaa saisi varmaan myös huomattavasti alas. Mutta ennen kuin sitä kannattaa tehdä, pitäisi tarkistaa laskussa käytetyt approksimaatiot.

  4. Koditon sanoo:

    Sellaisen vielä laitan, että onko fotinilla aaltofunktio joka määräisi sen koon, kuten piemeän aineen hiukkasen tapauksessa.

    1. Syksy Räsänen sanoo:

      On.

  5. Päivystävä fenomenologi sanoo:

    ”Osoittautui, että kun fotoni kulkee pimeän aineen hiukkasen läpi, pimeän aineen hiukkasen aiheuttama kaarevuus antaa fotonille massan, joka on sitä isompi mitä suurempi kaarevuus on.”

    No mutta eikö nykyajan tiedepopulistien pyhissä opinkappaleissa nimenomaan painoteta, ettei fotonilla ole mitään massaa? Toisaalta en tätä hämmennystä erityisemmin ihmettele, sillä eiväthän nämä suuria totuuksia laukovat tiedemaailman ylipapit osaa selvittää edes hiukkasten perusolemusta, vaikka aiheesta niin paavilliseen sävyyn rahvaalle saarnaavatkin. Paradoksien riivaama suhteellisuusteoria on ehkä se keskeisin ideologisuskonnollinen oppi, jota vain arvostetuin yläluokka kykenee lapsenomaisin kielikuvin selventämään, saaden kritiikittömät ihmismassat ihastelemaan luonnontieteellisen menetelmän saavutuksia.

    Mutta niin tai näin, ainoa varma pohja tieteenfilosofialle on jokin ehdottoman varma lähtökohta, eli pelkistetyimmillään inhimillinen tietoisuus. Kehoitankin siis herra kosmologia ja kumppaneita pohtimaan nöyrästi nykytietämyksen rajoja sekä saavuttamaan fenomenologisen reduktion, joka johdattaa kohden syvällisempää totuutta. Suurin arvo olisi tiedepopulisteille itselleen.

    1. Syksy Räsänen sanoo:

      Tällaisten näennäisten ristiriitaisten ilmausten taustalle on usein se, että fysiikkaa esittelevissä populaareissa esityksissä käytetään kieltä aina epätäsmällisesti, ja alan ulkopuoliselle voi olla vaikeaa suhteuttaa kahta yksinkertaistettua esitystä toisiinsa.

      Kirjoitin aiheesta hieman täällä: https://www.ursa.fi/blogi/kosmokseen-kirjoitettua/suureellinen-fantasiaeepos/

      Kun sanotaan, että fotonilla ei ole massaa, tarkoitetaan sitä, että vapaasti matkaavalla fotonilla, joka ei vuorovaikuta minkään kanssa, ei ole massaa. Sama pätee vaikkapa elektroneihin. Elektronit saavat massan, koska ne vuorovaikuttavat Higgsin kentän kanssa – voi myös sanoa, että ne käyttäytyvät kuten niillä olisi massa sen sijaan että niillä olisi massa.

      Vastaavasti fotonit saavat massan esimerkiksi kulkiessaan plasman tai metallin läpi. Philip Anderson itse asiassa löysi tämän ilmiön, joka on lähellä Higgsin mekanismia, ennen Higgsiä, ja joidenkin mielestä hänen olisi pitänyt saada osansa vuoden 2013 Nobelin palkinnosta Englertin ja Higgsin kanssa.

      Aiheesta lisää täällä: https://web.archive.org/web/20220526101420/http://www.tiede.fi/blogit/maailmankaikkeutta_etsimassa/perustuslakien_saatamisjarjestys

      Sofien ja minun löytämä ilmiö on tälle sukua.

      Suhteellisuusteoriassa ei ole paradokseja (eli sisäisiä ristiriitoja), tarkemmin ks. https://journal.fi/tt/article/view/41570

      Pyydän kirjoittamaan asiallisesti.

  6. Erkki Kolehmainen sanoo:

    ”Pyydän kirjoittamaan asiallisesti” on hieno toive, mutta voiko se toteutua, jos kirjoittajan lähtökohta on erilainen kuin Syksyn? Viittaan tällä päivystävän fenomenologin määritelmään, että ”ainoa varma pohja tieteenfilosofialle on jokin ehdottoman varma lähtökohta, eli pelkistetyimmillään inhimillinen tietoisuus.” Tässä suhteessa minulle maailman selittäjinä jumala ja pimeä aine/energia ovat samanarvoisia, sillä kummastakaan ei ole suoraa kokeellista havaintoa.

    1. Syksy Räsänen sanoo:

      Asiallisesti kirjoittaminen on erityisen tärkeää silloin kun lähtökohdat ovat erilaiset.

      Pimeä aine on sata vuotta vanha hypoteesi, joka on onnistuneesti selittänyt ja ennustanut suuren määrän erilaisia havaintoja, joita ei ole pystytty selittämään millään muulla tavalla. Se on siis erittäin onnistunut tieteellinen idea.

      Pimeä energia on teoreettisena ideana vielä vanhempi, vuodelta 1917, mutta havaintojen puolesta sitä on tarvittu vasta 90-luvulta alkaen. Se on selittänyt ja ennustanut oikein monia havaintoja (ei yhtä paljon kuin pimeä aine) yli 20 vuoden ajan. Idea voi olla oikein tai väärin, mutta se on tieteellisesti testattavissa.

  7. Martti V sanoo:

    Xenon tyyppiset kokeet taitaa rajata pimeän aineen 10 protonin massaan. Tämä tutkimus rajaa selkeästi alemmas ja jos on paikkansa pitävä, taitaa tehdä suorat hiukkashavainnot mahdottomaksi neutriinotaustakohinasta?

    1. Syksy Räsänen sanoo:

      XENONin tyyppiset kokeet, missä yritetään havaita pimeä aine suoraan, eivät rajaa pimeän aineen massaa yksin. Ne rajaavat sitä, miten raskas pimeän aineen hiukkanen voi olla, jos se vuorovaikuttaa tietyllä voimakkuudella tavallisen aineen kanssa.

      Niiden kannalta pimeän aineen massa voi olla miten iso tai pieni tahansa jos vuorovaikutus on tarpeeksi heikko – silloin hiukkasta ei kuitenkaan nähdä.

      Tämä gravitaatioon perustuva raja -jos se pitää paikkansa- ei riipu hiukkasen vuorovaikutuksen voimakkuudesta, ja pätee siis kaikkiin pimeän aineen hiukkasiin.

      Kevyiden pimeän ainen hiukkasten signaalin voi periaatteessa erottaa suoran havaitsemisen kokeissa nautriinoista suunnan perusteella. Neutriinothan tulevat enimmäkseen Auringosta, kun taas pimeä aine kulkee saman verran joka suuntaan. Ks. tämä suunniteltu koe (jonka herkkyys kyllä loppuu tuossa protonin massan kymmenesosan tienoilla): https://www.ursa.fi/blogi/kosmokseen-kirjoitettua/suunnistuksen-lahtokynnys/

      1. Martti V sanoo:

        Tosiaan vaikka pimeäaine tuntisi heikon vuorovaikutuksen, sitä ei tulla suoraan löytämään kosmisesta neutriinotaustasta enää tuolla massan ylärajalla. lman vuorovaikutusta havaiseminen on vielä mahdottomampaa ja kyse olisi eksoottisesta hiukkasesta . Steriili neutriinokaan ei taida enää olla vahva kandidaatti?

        1. Syksy Räsänen sanoo:

          Steriili neutriino on suosikkikandidaattini pimeäksi aineeksi. Se sopii vielä havaintoihin, ja on joitakin havaintoja, jotka saattavat olla siitä suora merkki. (Tai sitten ne saattavat, kuten monet aiemmat havainnot, selittyä tavallisilla tähtitieteellisillä lähteillä pimeän aineen sijaan.)

          Tarkemmin, ks.

          https://www.ursa.fi/blogi/kosmokseen-kirjoitettua/huippujen-laskeminen/

          https://www.ursa.fi/blogi/kosmokseen-kirjoitettua/vasemmalta-oikealle/

          https://www.ursa.fi/blogi/kosmokseen-kirjoitettua/kauneusvirheen-korjaaminen/

Vastaa

Sähköpostiosoitettasi ei julkaista. Pakolliset kentät on merkitty *


Matkakertomuksia

27.11.2022 klo 01.59, kirjoittaja
Kategoriat: Kosmokseen kirjoitettua , Kosmologia

Kesällä kuulin konferenssissa Jocelyn Bell Burnerin puheen, missä hän suositteli teosta The Sky is for Everyone. Nyt luin kirjan, ja se onkin kiinnostava.

Kirja alkaa luettelomaisella esittelyllä joistakin tähtitieteessä 1800-luvulta alkaen toimineista naisista. Sen varsinainen anti on 37 seuraavaa lukua, joissa kussakin yksi naistähtitieteilijä kirjoittaa urastaan. Heidän kertomuksensa on järjestetty väittelemisajankohdan mukaan vuodesta 1963 vuoteen 2010. Mukaan on valittu vain hyvin ansioituneita tutkijoita, ja kirja näyttää läpileikkauksen tähtitieteen kehitykseen ja läpimurtoihin viimeisten 60 vuoden ajalta.

Suurin osa kirjoittajista on työskennellyt ainakin osittain Yhdysvalloissa, mutta käsiteltyä tulee myös ainakin Iso-Britannian, Brasilian, Israelin, Etelä-Afrikan, Puolan, Japanin, Liettuan, Kiinan, Meksikon ja Intian tieteellistä yhteisöä ja kulttuuria.

Olen aiemmin maininnut, että naisfyysikoiden kertomukset omista kokemuksista valaisevat sitä miten sukupuoleen kohdistuvissa asenteissa ja säännöissä on tapahtunut kehitystä ja mitä on vielä korjattavana. Kirja kehystää näkyväksi sitä, miksi naiset ovat historiallisesti ja yhä aliedustettuina niin tähtitieteessä kuin muualla fysiikassa. (Lisää aiheesta täällä, täällä, täällä, täällä, täällä, täällä, täällä ja täällä.)

Kun Bell Burner oli väitöskirjaopiskelija, hän löysi vuonna 1967 yhdessä ohjaajansa Antony Hewishiä kanssa pulsarit. Toimittajat kysyivät Hewishiltä tieteestä, Bell Burnerilta taasen vyötärön, lantion ja poven mitoista sekä poikaystävien määrästä, ja pyysivät häntä avaamaan paidan nappeja valokuvia varten.

1960-luvulla Yhdysvalloissa saattoi käydä niin, että vaikka nainen palkattiin tutkijaksi, hänen saapuessaan yliopistoon kävi ilmi, että hänelle ei anneta työhuonetta tai makseta mitään. Palkkaa pyytäessä saattoi saada vastaukseksi, että ethän tarvitse rahaa muuhun kuin turkiksiin ja hajuvesiin, koska miehesi on töissä. Jos palkkaa saikin, se oli tuntuvasti pienempi kuin miehillä.

Yhdysvalloissa ei 1960-luvulla ollut tähtitieteestä väitelleille työpaikkailmoituksia. Hakemusten arvioimisen sijaan paikat jaettiin sen mukaan, kuka alaa johtavista miehistä soitti kenellekin suositellakseen omia opiskelijoitaan. Toisaalta tilannetta helpotti se, että avoimia työpaikkoja suhteessa valmistuneiden määrään oli enemmän kuin nykyään.

Sen lisäksi, että naisia ei pidetty pätevinä, myös avoimesti sanottiin, että heitä ei kannata palkata, koska naiset joko menevät naimisiin ja siksi lopettavat, tai eivät mene naimisiin ja aiheuttavat siksi harmia mieskollegoiden keskuudessa. Uhkaukset irtisanomisesta raskauden takia eivät olleet tavattomia.

Mielenkiintoinen vivahde syrjinnässä olivat ”nepotismin vastaiset” säännöt yliopistoissa, jotka kielsivät henkilökunnan perheenjäsenten palkkaamisen. Niitä käytettiin naisten palkkaamisen estämiseen, monet naistähtitieteilijät kun olivat naimisissa kollegan kanssa.

Avoin seksuaalinen häirintä tulee kirjassa esille, mutta myös se, miten mielestään mukavatkin tutkijat ovat saattaneet olla epämiellyttäviä. Roberta A. Humphreys kertoo, miten hänen vanhemmalla yhteistyökumppanillaan Allan Sandagella oli tapana aloittaa puhelunsa sanomalla ”hei prinsessa, Allan-setäsi tässä”. Humphreys kirjoittaa, että hänelle tuli mieleen prinsessa Leia ja Darth Vader. Toisaalta monet kirjoittajat etenkin alkupuolelta mainitsevat, miten tärkeää on ollut heitä tukeneiden ja tasa-arvoisesti kohdelleiden vanhempien miestutkijoiden tuki. Akateemisessa yhteisössä onkin vaikea edetä, sukupuolesta riippumatta, ilman korkeammassa asemassa olevien tukea.

Myös aviomiesten tuki lasten hankkimisen ja tieteellisen työn yhdistämisessä sekä laajemmin tulee vahvasti esille. Kertomuksissa alalle päätymisessä korostuu populaarien tiedekirjojen, scifin, innostavien ja asiantuntevien opettajien sekä kannustavien vanhempien merkitys.

Jotkut kirjoittajat sanovat, että eivät ole havainneet sukupuolestaan olleen heille tieteessä haittaa, tai jopa että siitä on ollut etua, koska he ovat erottuneet joukosta ja jääneet paremmin mieleen. Toiset toteavat, että he ovat olleet sokeita naisten kohtaamille ongelmille, koska eivät ole halunneet keskittyä niihin, ovat kasvattaneet paksun nahan ja karistaneet epämiellyttävät asiat mielestään.

Vaikka uudemmissa kirjoituksissa tulee esille vähemmän epäasiallista kohtelua kuin vanhemmissa, se korostuu enemmän. Tämä voi johtua osittain siitä, että se on tuoreemmassa muistissa, eikä ole yhtä hyväksytty osa työyhteisöä. Nuoremmalla sukupolvella tuntuu myös olevan syrjinnän hahmottamiseen ja vastustamiseen enemmän työkaluja. Loppupuolella tähtitieteilijöiden kirjon laajeneminen tuo mukaan rodullistamisen ongelmat.

On kiinnostava lukea erilaisista taustoista tähtitieteen eri aloille päätyneiden tutkijoiden kuvauksia tieteellisestä matkastaan. Heidän henkilökohtainen tarinansa kytkeytyy laajempaan yhteiskunnalliseen taustaan, joskus kirjan sortoon liittyvää aihepiiriä tahattomasti havainnollistaen. Paljon kertoo myös se, mitä ei sanota, ja itseään edistyksellisinä pitävät tutkijat saattavat kyseenalaistamatta omaksua käsityksiä yhteiskunnasta, jossa ovat kasvaneet.

Israelilainen Neta Bahcall esittää Israelin kolonialistiset sodat sotina henkiinjäämisestä. Israelilainen Dina Prialnik julistaa ”muurien kaatamisen” tärkeyttä yhteiskunnassa, vaikuttaen sokealta sille, että elää rotuerotteluun perustuvassa yhteiskunnassa. Hän ollut vararehtorina Tel Avivin yliopistossa, joka aktiivisesti tukee apartheidia ja auttaa ylläpitämään sotilasmiehitystä. Yhdysvaltalainen France Córdova hehkuttaa Israelin miehittämiä alueita osana Israelia rauhan nimessä. On vaikea uskoa, että keneltäkään hyväksyttäisiin kirjaan vastaavaa tekstiä Venäjän miehittämistä alueista.

Kirjassa on myös toisenlaisia näkökulmia sotaan ja rotusortoon. Judith (Judy) Gamora Cohen kirjoittaa osallistumisesta Vietnamin sodan vastaisiin mielenosoituksiin. Gillian (Jill) Knapp korostaa organisoimiensa vankiloissa pidettävien yliopistokurssien konkreettista merkitystä rasisminvastaiselle työlle Yhdysvalloissa. Patricia Anna Whitelock kertoo aktivismistaan poliittisten vankien puolesta apartheidin ajan Etelä-Afrikassa.

Henkilökohtainen ote tuo hyvin esille myös intohimon tieteeseen: sivuilta loistaa tutkimuksen jännitys, kuten myös saavutuksiin vaadittava panostus. Kirjoittajat ovat hyvin valikoitu joukko, eikä heidän kokemuksensa ole tyypillinen, vaan edustaa menestynyttä kärkeä. Kirja on positiivinen ja innostunut.

Niin kilpailu rahoituksesta kuin teleskooppien nimet tulevat lukujen edetessä tutuksi, ja tarinassa näkee tähtitieteen kentän kasvun eksoplaneettoihin, kosmiseen mikroaaltotaustaan ja gravitaatioaaltoihin asti. On hauska seurata, miten mahdottomina tai tyhjänpäiväisinä pidetyistä tutkimuskohteista tulee ensin juhlittuja läpimurtoja ja sitten arkea. Kehityksestä näkyy, miten tähtitieteessä on säilynyt vahva yhteys teorian ja kokeiden välillä. Hiukkasfysiikassahan tämä suhde on viime vuosikymmeninä heikentynyt.

Kirjasta tulee hyvin esille se, että –kuten Beatriz Barbuy omassa luvussaan kirjoittaa– tieteellinen työ muodostaa ison osan tekijänsä identiteettiä. Näissä tähtitiedettä eteenpäin vieneiden naisten kuvauksissa välittyy myös tutkijoiden identiteetin vaikutus tutkimukseen. Ne kertovat –hyvässä ja pahassa– tutkijoiden vahvoista siteistä, tieteen yhteisöllisyydestä ja tiedeyhteisön suhteesta yhteiskunnan kehitykseen, paljon laajemmin kuin yhdellä tieteenalalla ja 37 ihmisen työssä.

3 kommenttia “Matkakertomuksia”

  1. Erkki Kolehmainen sanoo:

    Vertailun vuoksi tässä yhteydessä kannattaa mainita Turun yliopiston tähtitieteen prof. Liisi Oterma, joka tanskalaisen kollegan mukaan vaikeni yhdellätoista kielellä!

    https://fi.wikipedia.org/wiki/Liisi_Oterma

  2. Erkki Kolehmainen sanoo:

    Liisi Oterman ansiolistalla on mm. 54 pikkuplaneetan löytäminen. Kun tuossa Syksyn listalla ei mainittu Venäjää eikä NL:a lainkaan, niin venäläinen Tamara Smirnova on löytänyt 135 pikkuplaneettaa tietääkseni Krimillä sijaitsevalla observatoriolla!

    https://en.wikipedia.org/wiki/Tamara_Smirnova

    1. Syksy Räsänen sanoo:

      Neuvostoliitto ja Venäjä ovat tosiaan kirjassa aliedustetut. Ainoa Neuvostoliitossa työskennellut astrofyysikko kirjassa on liettualainen Gražina Tautvaišienė.

Vastaa

Sähköpostiosoitettasi ei julkaista. Pakolliset kentät on merkitty *


Myös me, kierros 4/We too, round 4

17.11.2022 klo 23.12, kirjoittaja
Kategoriat: Kosmokseen kirjoitettua

(The case of Christian Ott has attracted international attention, so this post is both in Finnish and English. The English version is below the Finnish text. See also these three earlier posts on the topic.)

(Tämä on jatkoa kolmelle aiemmalle merkinnälle.)

Varsinais-Suomen käräjäoikeus antoi tänään päätöksensä minua ja kollegani ja ystävääni Till Sawalaa koskevassa oikeusjutussa. Meitä syytettiin törkeästä kunnianloukkauksesta ja törkeästä yksityiselämää loukkaavasta tiedon levittämisestä.

Kyse on tapahtumista tammi-helmikuussa 2018, jolloin astrofyysikko Christian Ott palkattiin Turun yliopistoon. Minä ja Sawala, muiden muassa, toimme esille Ottin häirintätaustaa hänen edellisessä työpaikassaan Caltechin yliopistossa, vastustimme häirintää ja arvostelimme päätöstä. Turun yliopisto perui palkkauksen. Ott haastoi oikeuteen asiassa mukana olleen Tukholman yliopiston sekä Turun yliopiston. Lisäksi hän teki tutkintapyynnön minusta ja Sawalasta, mikä johti prosessiin, joka kesti yli kolme vuotta, kaksi esitutkintakierrosta ja viisi oikeusistuntoa.

Nyt oikeus hylkäsi syytteet.

Turun Sanomat, Iltalehti, Ilta-Sanomat ja Helsingin Sanomat on kirjoittanut päätöksestä. Turun Sanomat kirjoitti myös oikeudenkäynnin kulusta asiallisesti, samoin Iltalehti. Helsingin Sanomatkin käsitteli asiaa, muun muassa otsikolla ”Häirintäsyytöksiä levitellyt kosmologi Syksy Räsänen pääsi ääneen oikeudessa”.

Tiedelehdessä Science Jeffrey Mervis kirjoitti viime viikolla oikeudenkäynnistä laajemmasta näkökulmasta, ottaen esille sen mitä se merkitsee häirintätapauksista puhumiselle. Tammikuussa 2016 Mervis oli ensimmäinen toimittaja, joka kirjoitti Ottin häirintätapauksesta Caltechissa.

Tänään julkaistussa tiedotteessamme minä ja Till Sawala käymme hieman läpi tapausta. Lisää yksityiskohtia löytyy kolmesta aiemmasta blogimerkinnästäni. Kommentoin tiedotteessa seuraavasti:

”On tervetullutta, että olemme viimein, yli kolme vuotta kestäneen prosessin jälkeen, saaneet vapauttavan tuomion. Toivottavasti tämä toimii ennakkotapauksena.

Kenenkään ei pitäisi pelätä sakkoja tai vankeustuomiota vain siksi, että he puhuvat häirintää vastaan, perustuen laajalti ja luotettavasti raportoituihin tosiseikkoihin. Pelkkä sen uhka voi tukahduttaa keskustelua, ja haitata häirinnän vastaista työtä.

Meillä oli taloudelliset resurssit, ja tiedeyhteisömme henkinen tuki, joilla vastata perättömiin syytteisiin meitä vastaan. Jos joku heikommassa asemassa oleva, kuten jatko-opiskelija, joutuisi tähän tilanteeseen, heille ei ehkä kävisi yhtä hyvin.”

Kunnianloukkausoikeudenkäyntien ongelmat eivät rajoitu tiedeyhteisöön. Tutkitusti Suomessa ei ole selvää linjaa sille, miten kunnianloukkaustapausten esitutkintaa suoritetaan ja millaisiin päätöksiin päädytään. Rikosilmoituksia kunnianloukkauksista myös käytetään häiriköinnin välineenä. Hyvin tunnetussa toimittaja Johanna Vehkoon tapauksessa käsittely kesti viisi vuotta ja päätyi korkeimpaan oikeuteen.

Meidän tapauksessamme ei ole syytä epäillä, etteikö Ott olisi kokenut, että hänelle on tehty vääryyttä. Mutta ottaen huomioon, että esille tuomamme tiedot oli laajalti raportoitu luotettavissa lähteissä, prosessin olisi silti pitänyt pysähtyä aiemmin. Kun tapauksen ensimmäinen syyttäjä päätti jättää syytteet nostamatta, apulaisvaltakunnansyyttäjä kumosi päätöksen koska ”syyttäjä oli viitannut yksinomaan kansainvälisiin medialähteisiin arvioimatta niiden luotettavuutta tarkemmin”.

On kohtuutonta, että maailman arvostetuimpien tiedejulkaisujen Naturen ja Sciencen raportoimia faktoja, joita mikään taho ei ole julkisesti kiistänyt, ei saisi mainita eikä ottaa keskustelun pohjaksi. Jos rima nostetaan näin korkealle, julkinen keskustelu häirinnästä ja muista kiistanalaisista asioista muuttuu mahdottomaksi.

Tältä kannalta on tärkeää, että oikeus päätöksessään toteaa, että vaikka ”häirintään ja syrjintään liittyvää tietoa on vastaajien toimesta ainakin jossain määrin järjestelmällisesti tuotu suuren ihmismäärän tietoon”, tämä ei ole ”kiellettyä saati rikosoikeudellisesti rangaistavaa”, vaan sitä voidaan päinvastoin ”pitää sellaisena yleisen edun mukaisena asiana, josta tulee voida avoimesti keskustella”. Oikeus toteaa lisäksi, että ”Ottista uutisoineiden medioiden luotettavuus huomioon ottaen varsin lähdekriittinenkin lukija on […] voinut luottaa kyseisen uutisoinnin oikeellisuuteen”.

Oikeus myös toteaa, että ”asialla on oletettavasti ollut merkitystä koko pienelle tiedeyhteisölle ja se on ollut yhteiskunnallisesti ja myös maailmanlaajuisesti merkityksellinen ja laajaa keskustelua herättänyt asia, mikä korostaa sananvapauden suojaa”.

Tapauksen oikeuskäsittely toi esille ongelmia häirinnän käsittelyssä myös tiedeyhteisön sisällä.

Turun yliopiston entisen rehtorin todistajanlausunto oikeudessa alleviivasi sitä, että yliopiston johto ei ollut huolissaan niinkään Ottin toiminnasta kuin mainehaitasta. Kuten olen aiemmin kirjoittanut, tällainen keskittyminen maineeseen on ongelmallista, koska siinä ohitetaan sen, mitä on todellisuudessa tapahtunut. Tämä voi johtaa siihen, että niin kauan kuin tosiseikat eivät tule julki, niistä ei piitata. Asiassa on myös toinen puoli: jos henkilöön kohdistuvat väitteet olisivat perättömiä, hänen työsopimuksensa purkaminen tai muut vastaavat toimet eivät olisi oikeutettuja.

Mainehaittaan keskittyminen saattaa myös motivoida peittelemään tapahtumia. Oikeudenkäynnissä tuli ainetodisteiden ja todistajanlausuntojen kautta esille se, miten osa henkilökunnasta Tuorlan observatoriossa, minne Ottia oltiin palkkaamassa, ei uskaltanut puhua asiasta. Yksi henkilökunnan jäsen kirjoitti tapahtumien aikaan kokevansa, että heitä on kielletty puhumasta medialle ja kuvaili ilmapiiriä sanomalla että ”olo on sellainen, että pitää olla valmis pakenemaan ikkunan kautta minä hetkenä hyvänsä”.

Turun yliopiston tähtitieteen emeritusprofessori Esko Valtaoja sanoi todistajanlausunnossaan oikeudessa, että hänen mielestään prosessi oli johdon kannalta täydellisesti epäonnistunut värväyspäätöksestä siihen, miten se kerrottiin henkilökunnalle, ja miten sen jälkeen toimittiin. Hän kommentoi, että menettelytapa, jossa ei oteta huomioon laitoksen henkilökuntaa ja ”vääristellään” yliopiston johdolle, miten suurin osa heistä muka odottaa innolla yhteistyötä Ottin kanssa ei ole hyväksyttävä.

Tuorlan observatorion johtaja Juri Poutanen oli yksi kahdesta henkilöstä, jotka ehdottivat Ottin palkkausta Turun yliopiston johdolle. Hän sanoi oikeudenkäynnissä, että vain yksi henkilökunnan jäsen oli henkilökuntatapaamisessa arvostellut palkkausta hänelle. Tämän jälkeen Ott oli tehnyt poliisille tutkintapyynnön kyseisestä henkilöstä, koska epäili hänen puhuneen Turun Sanomille kyseisestä tilaisuudesta ja Ottin palkkaamisesta. Poliisi kuulusteli henkilöä esitutkinnassa, mutta ei vienyt asiaa pidemmälle.

Poutanen oli palkkauksen aikoihin kirjoittanut, että hänen mielestään ei ole todisteita, että Ott olisi syyllistynyt häirintään. Hän oli syyttäjän todistajana oikeudessa. Oikeudenkäynnissä Poutaselle esitettiin Caltechin tutkimuksen tulokset, joiden mukaan Ott oli yksikäsitteisesti syyllistynyt sukupuoleen perustuvaan häirintään, ja kysyttiin pitävätkö ne hänen mielestään paikkansa. Poutanen totesi että ”minulle on aivan sama oikeastaan mitä siellä Caltechissa tapahtui”.

Astrofysiikassa on viime vuosina tullut esille korkean profiilin häirintätapauksia, joihin liittyy sukupuoleen perustuva tai seksuaalinen häirintä, viimeksi lokakuussa Leidenin yliopistossa Alankomaissa. On tyypillistä, että häirintään ei aluksi suhtauduttu vakavasti. Lausunnossaan Leidenin yliopiston johtokunta poikkeuksellisen avoimesti myönsi, että vaikka käytös oli huomattu, siihen ei puututtu tarpeeksi.

Häirintä on ongelma kaikilla aloilla ja kaikissa yhteisöissä. Se korostuu yhteisöissä, jotka ovat hyvin hierarkisia, joiden jäsenet kokevat yhteenkuuluvuutta ensisijaisesti niiden kanssa joilla on sama status, ja joissa korkeammassa asemassa olevat käyttävät merkittävää valtaa muihin. Tämä pitää paikkansa tiedeyhteisössä. Se että fysiikassa miesten osuus on isompi korkeammissa asemissa tekee alasta alttiimman sukupuoleen perustavalle ja seksuaaliselle häirinnälle, koska se on sukupuolittunutta.

Väitöskirjaohjaaja on ratkaisevassa asemassa jatko-opiskelijan tieteellisen uran kannalta. Ohjaajan ja ohjattavan suhde muistuttaa mestarin ja oppipojan suhdetta, ja on yleensä paljolti kahdenvälinen (tosin usein osana laajempaa tutkimusryhmää). Jatko-opiskelijat ovat haavoittuvassa asemassa myös siksi, että he ovat vasta aloittamassa tiedeyhteisöön nivoutumista.

Oikeusprosessin jälkeen tämä tapaus on minun ja Tillin osalta ohi. (En tosin tiedä, aikovatko syyttäjä tai Ott valittaa hovioikeuteen.) Samaa ei voi sanoa kaikista niistä, jotka ovat olleet tai ovat yhä häirinnän kohteena, tai joiden työpaikalla häirintään ja työntekijöiden hyvinvointiin suhtaudutaan välinpitämättömästi.

Päivitys (21/12/22): Syyttäjä eikä Christian Ott ei valittanut oikeuden päätöksestä, joten se on nyt lainvoimainen.

* * *

Today, the District Court of Southwest Finland delivered its judgment in the court case concerning me and my colleague and friend Till Sawala. We were accused of “aggravated defamation” and “aggravated dissemination of information that violates privacy”.

The case concerns events in January and February 2018, when astrophysicist Christian Ott was hired at the University of Turku. Me and Sawala, among others, spoke out about Ott’s harassment background and criticised the decision. The University of Turku cancelled the hire. Ott sued the University of Stockholm, which was involved in the hire, and the University of Turku. He also asked the police to investigate me and Sawala, leading to a process that took over three years, two rounds of preliminary investigation and five sessions in court.

Now the court dismissed the charges against us.

The newspapers Turun Sanomat, Iltalehti, Ilta-Sanomat and Helsingin Sanomat have written about the decision. The newspaper Turun Sanomat also wrote reasonable accounts of the trial (in Finnish), as did Iltalehti. Helsingin Sanomat also wrote about it, for example under the title “Cosmologist Syksy Räsänen who peddled harassment accusations got to speak in court”. In Science, journalist Jeffrey Mervis last week covered the case from a broader perspective, raising the issue of what it means for speaking out against harassment. In January 2016, Mervis broke the story of Ott’s harassment at Caltech.

In our press release today me and Till Sawala go over the case. More details can be found in my three previous blog entries. In the press release I comment as follows:

 “I welcome the acquittal after over three years of process. I hope this case will set a precedent.

No one should not have to fear fines or prison for simply speaking out against harassment based on widely and reliably reported facts. The threat alone can have a chilling effect that can set back work against harassment.

We had the financial resources, and support from our scientific community, to contest the baseless charges against us. If someone in a less secure position, such as a PhD student, were to be put in this situation, they might not fare so well.”

The problems of defamation trials are not limited to the scientific community. According to research, in Finland there is no clear policy on how preliminary investigations are conducted in defamation cases and what decisions are made. Criminal complaints about defamation are also used as a tool of harassment. In the well-known case of the journalist Johanna Vehkoo the proceedings took five years and went all the way to the Supreme Court.

In our case there is little reason to doubt that Ott felt himself to be wronged. But still, taking into account that the information we publicised had been widely reported in reliable sources, the process should have stopped earlier. After the first prosecutor declined to press charges, the deputy state prosecutor overturned the decision on the grounds that “the prosecutor had only referred to international media sources without evaluating their reliability in more detail”.

It is unreasonable that facts reported by the world’s premier science publications, which have not been publicly disputed by anyone, cannot be mentioned or taken as the basis of discussion. If the bar is set so high, public discussion of harassment and other controversial matters becomes impossible.

From this point of view it is important that the court notes in its decision that although “the defendants have at least to some measure systematically made known information about harassment and discrimination to large numbers of people”, this is not “forbidden nor punishable under criminal law”, but to the contrary it “can be considered a matter of public interest that people have to be able to discuss in public”.

The court also notes that “the matter has presumably been significant to the whole small scientific community and it has been socially and also a globally important and a widely discussed matter, which enhances the protection of the freedom of speech.” The court further observes that “taking into account the reliability of the media that have reported on Ott even a highly critical reader can […] have relied on the correctness of the reporting”.

The trial also exposed problems in how the scientific community deals with harassment.

The testimony of the former rector of the University of Turku highlighted that the university leadership was more concerned about damage to its reputation than what Ott had done. As I have earlier noted, such focus on reputation is problematic, because it treats what has actually happened as irrelevant. This can lead to ignoring the facts of the case as long as they do not become public. There is also another side to the matter: if allegations about a person were unfounded, terminating their contract or similar actions would be unjust.

Concentrating on damage to reputation can also motivate covering up events. Material evidence and testimony at the trial showed how some staff members at Tuorla Observatory, where Ott was to work, were too afraid to talk about the issue. One staff member wrote during the events that they felt they had been forbidden to speak to the media and described the atmosphere such that “you feel like you have to be ready to escape through the window at any moment”.

University of Turku astronomy professor emeritus Esko Valtaoja said in his testimony in court that in his view the process was a total failure on part of the leadership, from the hiring decision to how it was related to the staff, and what was done after that. He commented that not taking into account department staff and giving a distorted picture to the university leadership that most of them supposedly eagerly await working with Ott is not an acceptable way to handle matters.

Tuorla Observatory Director Juri Poutanen was one of the two people who proposed Ott’s hire to the university leadership. At the trial, he said that at a staff meeting only one member of staff had criticised Ott’s hire to him. After this Ott had made a criminal complaint about the staff member in question, suspecting him of having talked to the media about the meeting and Ott’s hire. The police questioned the person during preliminary investigation, but did not take the matter further.

During the hire Poutanen had written that in his view there is no evidence that Ott harassed anyone. At the trial Poutanen was presented with the findings of the Caltech investigation, according to which there was “unambiguous gender-based harassment of both graduate students” by Ott and asked whether he thinks they are correct. Poutanen replied that in his view ”it really makes no difference what happened at Caltech”.

Over the last few years, high-profile harassment cases that include sexual or gender-based harassment have come to light in astrophysics, the latest in October at Leiden University in the Netherlands. It is typical that harassment was not taken seriously at first. In its statement, Leiden University executive board admitted with rare candour that although the behaviour was noticed, not enough was done about it.

Harassment is a problem in all fields and communities. This is particularly so in communities that are very hierarchical, whose members feel a sense of camaraderie primarily with others of the same status, and where those in higher positions have significant power over others. This is the case in the scientific community. The fact that in physics there are more men in higher positions makes the field more prone to sexual and gender-based harassment, which is gendered.

The PhD supervisor plays a key role in a PhD student’s scientific career. The relationship between a supervisor and PhD student is reminiscent of the relationship between master and disciple, and is usually mostly a relationship between two people (although commonly in the context of a larger research group). PhD students are vulnerable also because they are just beginning to integrate into the scientific community.

After the legal process is over, this case will be over for me and Till. (Although I don’t know whether the prosecutor or Ott are going to appeal.) The same cannot be said for all those who have been or continue to be harassed, or in whose workplace harassment and employee wellbeing are given short shrift.

Update (21/12/22): Neither the prosecutor nor Christian Ott appealed, so the court’s decision is now final.

14 kommenttia “Myös me, kierros 4/We too, round 4”

  1. Human Rights Watch sanoo:

    Mitäköhän yleisen edun mukaista asiaa se ajaa, että henkilöä jahdataan ja ahdistellaan ympäri maailmaa kuin jotakin totalitaarisesta diktatuurista paennutta toisinajattelijaa? Ja millaista maolaista julkisia katumista Ottin pitäisi harjoittaa, että hän rehabilitoituisi ristiinnaulitsevan aktivistilauman silmissä? Suomalaisen oikeustajun perusteella Ottin sukupuoleen kohdistama häiritsevä käytös oli lisäksi erittäin vähäpätöistä. Eikö tällainen kampanja loukkaa jo oikeutta vapaaseen ammatinharjoittamiseen?

    1. Syksy Räsänen sanoo:

      Ottia ei ole jahdattu eikä ahdisteltu.

      Ammatilliseen väärinkäytökseen syyllistyneiden henkilöiden paluusta työyhteisöön kirjoitin aiemmin näin (https://www.ursa.fi/blogi/kosmokseen-kirjoitettua/myos-meus-too/): ”edellytyksenä on se, että hän ymmärtää tehneensä väärin ja ottaa vastuun tekemästään vahingosta”.

      Saman merkinnän tämä teksti sopii sen arvioimiseen, mitä Ott teki ja mikä sen vaikutus oli:

      ”sitä ei voi luotettavasti tehdä kuulemalla pelkästään häiritsijäksi todettua henkilöä ja hänen valitsemiaan tahoja. Jokainen tietysti haluaa esittää asiat itselleen parhain päin, ja ihmiset mielellään puolustavat kavereitaan. Siksi on tärkeää, että häirintäsyytöksiä ja muita kiistanalaisia asioita selvittävät riippumattomat tahot -kuten Caltechin komitea-, jotka kuulevat kaikkia asianosaisia ja tutustuvat kaikkeen dokumentaatioon.”

      Caltechin selvityksen mukaan Ottin häirintä ei ollut ”erittäin vähäpätöistä”.

      Caltechin tapauksen yksityiskohdista voi lukea merkinnän https://www.ursa.fi/blogi/kosmokseen-kirjoitettua/myos-me-kierros-3-we-too-round-3/ linkeistä, jotka löytyvät etsimällö sanat ”Caltechin tapauksen yksityiskohdista”.

      Mitä ammatinharjoittamiseen tulee, saman merkinnän kommenteissa kirjoitin: ”Kenelläkään ei ole nimenomaista oikeutta saada tutkijanpaikkaa, sen enempää kuin mitään muutakaan tiettyä työpaikkaa.”

      1. Periaatteessa ei liity aiheeseen, mutta:

        ”Jokainen tietysti haluaa esittää asiat itselleen parhain päin.” En ole samaa mieltä. Monet ihmiset pyrkivät puolueettomuuteen ja rehellisyyteen kaikissa asioissa, myös itseä koskevissa. Jokainen -sanan käytössä on hyvä olla varovainen.

        1. Syksy Räsänen sanoo:

          Saatat olla oikeassa.

    2. Pervert Rights Watch sanoo:

      Human Rights Watch, kuka mielestäsi todennäköisimmin huutaa ahdistelijoiden oikeuksien puolesta? Uhrit, sivusta katsojat vai kenties TOISET samanhenkiset, joille ahdistelu on vain rohkeaa lähestymistä, johon ristiinnaulitsijat eivät kykene?
      Jatkaisimmeko keskustelua omilla nimillämme tai lopettaisimmeko ahdistelijoiden puolustamisen?

      1. Syksy Räsänen sanoo:

        Kiitos kommentista. Pidetään kaikki keskustelu asiallisena.

  2. Erkki Tietäväinen sanoo:

    Tässä on herännyt seuraava kysymys:

    Syksy Räsänen ja Till Sawala perustivat kantansa siihen virheelliseen olettamaan, että Christian Ott oli syyllistynyt seksuaaliseen häirintään. Tätä he levittivät myös julkisuudessa eteen päin tunnetuin seurauksin. Entä jos heidän tietoonsa olisi alun perinkin tullut, että kyseessä oli ”vain” sukupuoliperusteinen häirintä? Olisiko Räsäsen ja Sawalan menettely ollut sama ja olisiko Ottin palkkaaminen Turun Yliopistoon silloinkin tullut peruutetuksi?

    1. Syksy Räsänen sanoo:

      Emme perustaneet kantaamme siihen. Meidän kannaltamme ei ole oleellista, oliko Ottin toiminta seksuaalista häirintää vai sukupuoleen perustuvaa häirintää. (Terminologia muuten ei ole vakiintunutta, ja noita termejä usein käytetään sekaisin.)

      Till Sawala ei ole julkisuudessa sanonut Ottin syyllistyneen seksuaaliseen häirintään. Minä mainitsin niin julkisesti kolmessa twiitissä. Lisäksi sanoimme niin muiden Turun yliopiston rehtorille lähetetyssä kirjeessä, jota emme ole julkistaneet. Vaihdoimme ilmaisuun sukupuoleen perustuva häirintä, koska se on kiistaton ja tässä tapauksessa tarkempi.

      Ottamatta kantaa siihen, oliko Ottin toiminta seksuaalista häirintää mainittakoon, että Ottin todettiin Caltechissa rikkoneen yliopiston seksuaalisen häirinnän vastaisia linjauksia, ja NASA:n ja Kansallisen tiedesäätiö NSF:n selvitys Caltechin menettelytavoista käsitteli hänen tapaustaan esimerkkinä seksuaalisesta häirinnästä.

      1. Cargo sanoo:

        ”Meidän kannaltamme ei ole oleellista, oliko Ottin toiminta seksuaalista häirintää vai sukupuoleen perustuvaa häirintää.”

        Ehkä sen pitäisi olla, sillä vain toinen kyseisistä häirinnän muodoista on Suomen rikoslaissa kriminalisoitu. Lienee myös ilmeistä, että julkinen kampanja sai aikaan sen, että Turun yliopisto irtisanoi Ottin. Amerikassa lähetettyjen viattomien runojen ei pidä vaikuttaa Suomessa tapahtuviin rekrytointeihin.

        Itse olen ymmärtänyt myös niin, että Caltech-kohussa osallisena ollut, silloin parikymppinen ja keskimääräistä viehättävämpi, neiti Kleiser sai potkut heikon tieteellisen suoritustason vuoksi, ei siis mistään ihmissuhdedraamasta johtuen. On myös osoittautunut, ettei neiti Kleiser pärjännyt akateemisessa maailmassa. Ehkäpä Ottin olisi pitänyt laittaa peli poikki jo paljon aikaisemmin, jolloin kostonhalulta oltaisiin vältytty. Voisin lyödä myös vetoa, että ohjaajapalavereissa tällä neiti Kleiserilla oli paidan ylin tai toinenkin nappi auki.

        1. Syksy Räsänen sanoo:

          On tavallista, että sukupuoleen perustuva ja seksuaalinen häirintä esitetään sen kohteiden syyksi. On myös tavallista, että niistä esitetään virheellisiä väitteitä.

          Tasa-arvolaissa on kielletty seksuaalinen häirintä ja sukupuoleen perustuva häirintä samassa pykälässä: https://finlex.fi/fi/laki/ajantasa/1986/19860609

          Asiasta voi lukea tarkemmin sukupuolentutkimuksen senioriprofessori Liisa Husun asiantuntijalausunnosta, joka oli yksi todiste oikeudenkäynnissä: https://www.mv.helsinki.fi/home/syrasane/Husu_lausunto.pdf

          Tämä on sinänsä toissijaista, että emme väittäneet, että Ottia olisi asetettu syytteeseen rikoksesta. Emme käsitelleet Ottin tekoja rikoksena, vaan vakavana ammatillisena väärinkäytöksenä.

          Mitä tulee Ottin toiminnan yksityiskohtiin, niissä ei ollut kyse vain ”viattomista runoista”: yksityiskohdista voi lukea merkinnän https://www.ursa.fi/blogi/kosmokseen-kirjoitettua/myos-me-kierros-3-we-too-round-3/ linkeistä, jotka löytyvät etsimällä sanat ”Caltechin tapauksen yksityiskohdista”.

          Tässä ote Caltechin tutkimuksen johtopäätöksistä (myös osa oikeudenkäynnin todistusaineistoa): https://www.mv.helsinki.fi/home/syrasane/Ott_conclusions.png

          Kleiser teki väitöskirjansa loppuun Caltechissä, väiteltyään hän siirtyi tutkijaksi NASAn Jet Propulsion Laboratoryyn.

  3. Jari Toivanen sanoo:

    Olet, Syksy, toiminut mielestäni ihan oikein, ja kiitos sinulle siitä.

    1. Syksy Räsänen sanoo:

      Kiitos.

  4. Metoo sanoo:

    Jotenkin tulee mieleen metoo kampanjat ja syylliseksi tuomitseminen ennen oikeaa oikeutta. Näin ei pitäisi koskaan mennä että muut tuomitsevat ennen oikeaa oikeutta.

    1. Syksy Räsänen sanoo:

      Monia häirintätapauksia selvitetään muuten kuin oikeusistuimessa. Tässä tapauksessa Caltechissa tehtiin yliopiston sisäinen selvitys.

Vastaa

Sähköpostiosoitettasi ei julkaista. Pakolliset kentät on merkitty *

Taivaan merkit

31.10.2022 klo 13.16, kirjoittaja
Kategoriat: Kosmokseen kirjoitettua , Kosmologia

Mainitsin viime kuussa, että kosmisen mikroaaltotaustan perusteella tiedämme, että pimeä aine (jos sitä on olemassa) on tähtiä vanhempaa. Merkinnän kommenteissa pyydettiin avaamaan sitä, miten kosmisesta mikroaaltotaustasta voi päätellä tällaisia asioita. Tämä onkin hauska aihe.

Varhaisina aikoina maailmankaikkeuden aine oli hiukkaskeittoa. Kun avaruus laajenee, keiton lämpötila laskee. Kun maailmankaikkeus saavutti 380 000 vuoden iän, lämpötila laski alle 3 000 kelvinin. Tällöin keiton valohiukkasten energia ei enää riittänyt atomiytimien ja elektronien välisen sidoksen rikkomiseen, joten ne yhtyivät atomeiksi.

Tätä ennen valo, elektronit ja ytimet olivat tiukasti kytköksissä, kun valo poukkoili elektronien sähkövarauksista, ja maailmankaikkeus oli läpinäkymätön. Atomit ovat sähköisesti neutraaleja, joten niiden muodostuttua valo ei juuri vuorovaikuta aineen kanssa, ja on siitä lähtien matkannut jokseenkin esteettä halki maailmankaikkeuden.

Tämä valo tunnetaan nimellä kosminen mikroaaltotausta. Se on kirjaimellisesti valokuva maailmankaikkeudesta 380 000 vuoden iässä, ja näyttää tältä:

Kosminen mikroaaltotausta. (Lähde: ESA:n Planck-tutkimusryhmä.)

Kuvassa näkyy taivas mikroaaltoaallonpituudella, kun on poistettu Linnunradasta ja joistakin muista lähellä olevista lähteistä tuleva säteily. Punakeltaiset alueet ovat kirkkaampia ja siniset himmeämpiä; erot ovat muutaman sadastuhannesosan kokoisia. Näistä taivaan merkeistä voi lukea, millainen maailmankaikkeus oli nuorena.

Suoraviivaisin päätelmä on se, että valon ja tavallisen aineen tiheys oli 380 000 vuoden aikaan sama kaikkialla sadastuhannesosan tarkkuudella. Tarkempaa tietoa saa tutkimalla sitä, miten täplien kirkkaus riippuu niiden koosta. Vältän blogissa kuvaajien käyttämistä, mutta tätä en malta olla laittamatta:

Kosmisen mikroaaltotaustan täplien kirkkauden ja koon suhde. Pisteet ovat Planck-satelliitin mittaustuloksia, käyrä on teorian paras ennuste. (Lähde: ESA:n Planck-tutkimusryhmä.)

Kuvassa on x-akselilla täplän koko taivaalla, ja y-akselilla se, paljonko kirkkaus poikkeaa keskiarvosta. Isoimmat täplät ovat 90 asteen kokoisia; Planck pystyy erottamaan pienimmillään vajaan asteen kymmenesosan kokoisia täpliä.

Kirkkaimpia ovat noin asteen kokoiset täplät. Tästä voi päätellä sen, miten nopeasti maailmankaikkeus laajenee. Sitä varten pitää tietää, miten täplät syntyvät.

Kun keskivertoa tiheämmät alueet varhaisina aikoina gravitaation takia tiivistyivät, valon paine työnsi niitä takaisin, mikä johti vuoroittaiseen tiivistymiseen ja harventumiseen. Edestakainen liike synnytti aaltoja, kuin järven pintaa vatkaava käsi. Aallot matkasivat nopeudella joka on noin puolet valonnopeudesta. Vanhimmat aallot olivat 380 000 vuoden aikaan ehtineet matkata 400 000 valovuotta, nuoremmat vähemmän. (Kuljettu matka on isompi kuin maailmankaikkeuden ikä kertaa nopeus, koska avaruus laajenee.)

Kappaleen kulmakoko taivaalla on sen pituus jaettuna sen etäisyydellä: mitä pienemmältä kappale näyttää, sitä kauempana se on. Kun siis tiedämme aaltojen pituuden ja kulmakoon, voimme päätellä kuinka kaukaa kosminen mikroaaltotausta on nykypäivään asti matkannut. Jos valon ja aineen eron hetkeä siirtäisi kauemmas tai lähemmäs, niin kaikkien täplien koko taivaalla muuttuisi tasaisesti. Tästä voi mitata etäisyyden tarkasti.

Koska tämä etäisyys riippuu siitä, miten maailmankaikkeus on laajentunut, kosmisesta mikroaaltotaustasta voi päätellä maailmankaikkeuden laajenemisnopeuden. Kosmologian tämän hetken merkittävin ristiriita ennusteiden ja havaintojen välillä onkin se, että tällä tavalla saa eri tuloksen kuin mittaamalla laajenemisnopeuden suoraan siitä, miten nopeasti lähellä olevat galaksit meistä etääntyvät.

Entäpä se pimeä aine? Aaltojen pituus taivaalla kertoo vain etäisyyden, mutta niiden korkeudesta voi lukea monta seikkaa. Mitä vahvemmin tiheiden alueiden gravitaatio varhaisina aikoina vetää ainetta puoleensa, eli mitä enemmän massaa niissä on, sitä voimakkaampia aallot ovat. Toisaalta näkyvä aine (eli elektronit ja atomiytimet) törmäilee koko ajan valoon, mikä hidastaa sen liikkeitä kitkan tavoin, ja vaimentaa aaltoja. Pimeällä aineella ei ole tällaista ongelmaa.

Mitä enemmän on pimeää ainetta, sitä korkeampi on pisimmän aallon aallonhuippu, ja mitä enemmän on näkyvää ainetta, sitä matalampi se on. Yhdestä huipusta ei siis voi päätellä erikseen pimeän aineen ja tavallisen aineen määrää, koska niitä molempia sopivasti kasvattamalla korkeus pysyy samana.

Mutta pimeä aine ja näkyvä aine vaikuttavat eri tavalla kuvassa näkyviin eri huippuihin. Kuvassa ei ole erotettu aallonharjoja ja -pohjia: siinä näkyy vain paljonko kirkkaus poikkeaa keskiarvosta, ei onko alue keskivertoa kirkkaampi vai himmeämpi. Joka toinen huippu vastaa itse asiassa aallonharjaa ja joka toinen aallonpohjaa.

Näkyvän aineen kitka syventää aallonpohjia ja laskee aallonhuippuja, kun taas pimeän aineen gravitaatio vahvistaa molempia. Niinpä ottamalla huomioon sekä ensimmäisen että toisen huipun korkeuden voi päätellä sekä pimeän aineen että näkyvän aineen tiheyden. Huippujen korkeuksien suhteesta voi lukea, että pimeää ainetta on noin viisi kertaa niin paljon kuin näkyvää ainetta.

Pimeä aine esitettiin alun perin selittämään sitä, miksi näkyvä aine galakseissa ja galaksiryppäissä liikkuu nopeammin kuin mitä sen oma gravitaatio pystyy selittämään. Tarvittiin ainetta, jota ei voi nähdä, eli joka ei juuri vuorovaikuta valon kanssa. Vapaus valosta osoittautui sittemmin avaimeksi myös kosmisen mikroaaltotaustan täplien ymmärtämiseen. Tämä on hyvä esimerkki siitä, miten oikeansuuntaiset ideat ratkaisevat myös uusia ongelmia ilman erillistä säätämistä – eli tekevät onnistuneita ennustuksia.

Ainoa vaihtoehto pimeälle aineelle on se, että gravitaatio käyttäytyisi eri tavalla kuin mitä yleinen suhteellisuusteoria ennustaa. On kuitenkin vaikea selittää, miksi kosmisen mikroaaltotaustan muodostumisen aikaan gravitaatio olisi kuusi kertaa odotettua vahvempi, mutta Aurinkokunnassa ei ole nähty poikkeamia yleisen suhteellisuusteorian ennusteista, vaikka niitä on mitattu sadastuhannesosan tarkkuudella. Yksikään ehdokas uudeksi gravitaatioteoriaksi ei ole pystynyt edes jälkikäteen selittämään kosmisen mikroaaltotaustan kaikkien huippujen korkeuksia, saati ennustamaan niitä.

Kosmisen mikroaaltotaustan analyysi on oikeasti monimutkaisempaa kuin vain huippujen korkeuksien ja aineen tiheyksien vertaaminen. Mikroaaltotaivaan merkeistä voi lukea fotonien ja neutriinojen tiheyden, testata kosmisen inflaation ennustetta siitä, millainen aaltojen lähteinä toimivien ylitiheiden alueiden jakauma on, ja paljon muuta.

Kosminen mikroaaltotausta on ehkä antoisin yksittäinen kosmologinen havainto: se sisältää paljon tietoa, sitä voidaan mitata tarkasti, ja sen teoreettinen tarkastelu on suoraviivaista. On paljon helpompi mallintaa pieniä aaltoja kaasussa kuin vaikkapa törmääviä mustia aukkoja.

Seuraavaksi halutaan mitata tarkemmin kosmisen mikroaaltotaustan fotonien polarisaatiota, eli sitä, mihin suuntiin ne värähtelevät. Tuloksia on odotettavissa kymmenen vuoden kuluessa japanilaiselta satelliitilta LiteBIRD sekä kansainvälisiltä maanpäällisiltä teleskooppihankkeilta Simons-observatorio ja CMB-S4.

27 kommenttia “Taivaan merkit”

  1. Boris the rat sanoo:

    hyvä luento!

    kysymys; voiko maailmankaikkeuden laajentuminen , johtua (mahdoliisesti) kauempana olevien universumien vetovoimalla?

    1. Syksy Räsänen sanoo:

      Maailmankaikkeus on kaikki aika ja avaruus, mitä on olemassa. Avaruuden laajenemisesta, ks. http://www.tiede.fi/artikkeli/blogit/maailmankaikkeutta_etsimassa/rajaton_kasvu

      1. Boris the rät sanoo:

        mutta eikö näitä multiversumi hommia ole esitetty?

        1. Syksy Räsänen sanoo:

          On, mutta niissä yleensä ”muilla maailmankaikkeuksilla” on kullakin oma erillinen aika-avaruutensa, joka ei ole yhteydessä meidän aika-avaruuteemme. Joskus sanalla ”maailmankaikkeus” myös viitataan oman avaruutemme kaukana toisistaan oleviin osiin.

          Multiversumi-ideasta tarkemmin, ks.

          https://www.ursa.fi/blogi/kosmokseen-kirjoitettua/kaikenlaisia-selityksia/

          1. Boris the Rat sanoo:

            Ja nämä eivät voi vetovoimallaan laajentaa omaa kaikkeuttamme?

          2. Syksy Räsänen sanoo:

            Avaruuden laajenemisen syynä ei ole se, että kappaleet vetävät toisiaan puoleensa.

          3. Boris the rat sanoo:

            luulin että laajentumisen syytä ei vielä tiedetty?

          4. Syksy Räsänen sanoo:

            On tiedetty sata vuotta, vuodesta 1922 asti. Siitä, miksi laajeneminen on viime aikoina kiihtynyt, ei tosin ole varmuutta.

  2. hölmö sanoo:

    voiko universumia laajentavuminen johtua toisten universumien vetovoimalla?
    vai onko mahdollista että me itse sattumalta sijaitsemme laajentuvassa kohtaa universumia?

    1. Syksy Räsänen sanoo:

      Maailmankaikkeus on kaikki aika ja avaruus, mitä on olemassa. Avaruuden laajenemisesta, ks. http://www.tiede.fi/artikkeli/blogit/maailmankaikkeutta_etsimassa/rajaton_kasvu

  3. Eusa sanoo:

    Eikö maaimankaikkeuden sijaan olisi luontevinta puhua yksinkertaisesti kaikkeudesta?

    1. Syksy Räsänen sanoo:

      Miksipä ei.

  4. Martti V sanoo:

    Taustasäteilyssä on havaittu selkeästi kylmempi alue tai ehkä useampia. On esitetty, että se voisi olla jälki kosketuksesta toiseen universumiin. Voidaanko tällaiset ajatukset tyrmätä?

    1. Syksy Räsänen sanoo:

      Kyseinen ”kylmänä täplänä” tunnettu alue ei itse asiassa ole poikkeuksellisen kylmä. Kyseessä saattaa olla pelkkä tilastollinen sattuma.

      Mainitsemasi selitys täplälle viittaa ideaan, että kosmisen inflaation aikana olisi syntynyt kuplia, joiden törmäyksestä olisi jäänyt jälki inflaatiota ajavaan kenttään, jonka kosminen mikroaaltotausta sitten perii. Onhan se mahdollista, mutta täplä ei ole tilastollisesti niin harvinainen, että se antaisi paljon tukea tällaiselle tapahtumalle.

  5. Helena Othman sanoo:

    Jotenkin kiehtovaa ajatella, että siinä hiukkassopassa alussa oli ns kaikki, siis jos ajattelee ikäänkuin nykyhetkestä taaksepäin. Ajan, liikkeen, lämmön jne lisäksi.
    Eikä siinä keitossa ilmeisesti ollut mitään mille ei löytynyt käyttöä.

    1. Syksy Räsänen sanoo:

      Hieman aineen muodonmuutoksista tässä:

      https://www.ursa.fi/blogi/kosmokseen-kirjoitettua/tuhoutuuko-kaikki/

  6. Boris the Rat sanoo:

    sen verran vielä pimeästä aiheesta;

    koska painovoima ajatellaan usein avaruuden kaareutumisena; onko avaruutta laajentavan pimeän aineen aiheuttama kelmu aika-avaruuteen ”negatiivinen”? pahoittelut kysymystulvasta

    1. Syksy Räsänen sanoo:

      Mitä tarkoittaa tässä ”kelmu” ja ”negatiivinen”?

      1. Boris the Rat sanoo:

        normaali massa esitetään aika-avaruuden kaareutumisena montulle ”alaspäin” – näin ollen pimiä aine kaareuttaa aika-avaruutta ”ylöspäin” satulan tapaan?

        1. Syksy Räsänen sanoo:

          Tällaista aika-avaruuden kaarevuutta havainnollistavaa kuvaa ei voi katsoa noin kirjaimellisesti. Aika-avaruuden kaarevuudella on oikeasti 20 eri suuntaa, ei vain yksi.

          Mutta tämä vastannee kysymykseesi: pimeän aineen gravitaatio on aivan samanlaista kuin tavallisen aineen. Niiden vaikutus avaruuden laajenemiseen ja kappaleiden välisene näennäiseen vetovoimaan on sama.

          1. Boris the Rat sanoo:

            anteeksi; tarkoitin pimeää energiaa

  7. miguel sanoo:

    Sellainen kysymys, että kun alku-universumi muuttui näkyväksi fotoneille, niin jos kaikkeus ei laajene valonnopeudella, vaan äärellisellä nopeudella, niin jossain vaiheessa fotonit ilmeisesti saavuttavat/ovat saavuttaneet universumin reunan. Mihin ne sen jälkeen etenevät? Neuriinot ilmeisesti saavuttaisivat laajenevan reunan jo aiemmin. Tietysti, jos universumi on rajaton tai ääretön, niin ei olisi mitään reunaa, jonka saavuttaa.

    1. Syksy Räsänen sanoo:

      Ei tiedetä onko maailmankaikkeus äärellinen vai ääretön, mutta jos se on äärellinen, se on rajaton. Fotonit kulkevat joka suuntaan.

      Avaruuden laajenemista ei mitata nopeuden yksiköissä. Mutta jos laajeneminen hidastuu, niin valo tosiaan saavuttaa ajan kuluessa mielivaltaisen etäisiä pisteitä. Jos laajeneminen kiihtyy, näin ei tapahdu.

      Tarkemmin avaruuden laajenemisesta, ks.

      https://www.tiede.fi/blogit/maailmankaikkeutta_etsimassa/rajaton_kasvu

      1. Jani sanoo:

        Kun kerta maailmankaikkeudella on alku ja siitä on äärellinen määrä aikaa, niin kuinka maailmankaikkeus voisi olla ääretön?

        1. Syksy Räsänen sanoo:

          Miksipä ei? Jos maailmankaikkeus on ääretön, se on ollut koko olemassaolonsa ajan ääretön.

        2. Kyllä, havaittavalla eli meille näkyvällä maailmankaikkeudella on alku josta on äärellinen määrä aikaa, ja havaittava maailmankaikkeus on äärellinen, ja tästä on yksimielisyys.

          Kosmisen horisontin takaa meillä ei ole havaintoja. Voidaan ajatella että olisi yksinkertaisinta jos maailmankaikkeus jatkuisi siellä suurena tai peräti äärettömänä, ehkä.

          Tietoamme rajoittavat vaikeus nähdä varhaisiin ajanhetkiin (inflaation aiheuttama diluutio ym.) ja kosminen horisontti. Ja jos käytetään kvanttimekaniikan monimaailmatulkintaa, niin myös siihen liittyvä ”kvanttihorisontti”, jonka takana ovat multiversumin ne haarat jotka eivät ole meille makroskooppisesti totta, eli jotka (löysästi sanoen) eivät interferoi konstruktiivisesti meistä katsoen. Eli noin kolme horisonttia.

          1. Syksy Räsänen sanoo:

            Selvennykseksi lukijoille, että ”havaittava maailmankaikkeus” tässä tarkoittaa aluetta, josta meille on ehtinyt tulla signaaleja.

Vastaa

Sähköpostiosoitettasi ei julkaista. Pakolliset kentät on merkitty *


Lomittuneilla fotoneilla

4.10.2022 klo 21.31, kirjoittaja
Kategoriat: Kosmokseen kirjoitettua

Ruotsin kuninkaallinen tiedeakatemia ilmoitti tänään, että vuoden 2022 Nobelin fysiikan palkinnon saavat Alain Aspect, John F. Clauser ja Anton Zeilinger ”lomittuneilla fotoneilla tehdyistä kokeista, jotka osoittivat Bellin epäyhtälön rikkoutumisen ja olivat kvantti-informaatiotieteen edelläkävijöitä”. Kuten tavallista, tiedeakatemia julkaisi erikseen suurelle yleisölle ja fyysikoille suunnatut taustoitukset.

Fyysikot tykkäävät arvella Nobelin palkintoja omalle alalleen, mutta olen vuosia kuullut niidenkin, jotka eivät työskentele kvanttimekaniikan perusteiden ja kvantti-informaation parissa kyselevän koska siitä myönnetään Nobelin palkinto Alain Aspectille ja muille avaintutkijoille.

Nyt palkitun tutkimuksen ytimessä on se, miten kvanttimekaniikka eroaa klassisesta mekaniikasta, ja samalla arkiajattelusta. Kvanttimekaniikan mukaan todellisuus ei ole määrätty: asioilla on todennäköisyydet olla eri tavoin, sen sijaan että ne olisivat yhdellä tavalla.

Erwin Schrödinger, yksi kvanttimekaniikan löytäjistä, päätti havainnollistaa kvanttimekaniikan tätä piirrettä ajatuskokeella missä on kissa, mikä on varmistanut sen, että idea on levinnyt laajalle. Schrödingerin kissa on sekoituksessa kuollutta ja elävää, olematta kumpaakaan. Schrödinger esitti ajatuskokeen vuonna 1935 osoittaakseen, että koska johtopäätös kissan tilasta on outo, kvanttimekaniikan pitää olla puutteellinen. Nykyään asia nähdään toisin päin: koska kvanttimekaniikka pitää niin suurella tarkkuudella paikkansa, todellisuus on outo.

Kaksoisrakokokeessa on kokeellisesti mitattu jäljet siitä, että hiukkasta kvanttimekaniikassa kuvaava todennäköisyysaalto samaan aikaan kulkee eri reittejä, sen sijaan että hiukkasella olisi määrätty rata. On kuitenkin esitetty niin kutsuttuja piilomuuttujateorioita, joiden mukaan hiukkasten tila on itse asiassa koko ajan määrätty, me vain emme tiedä mikä se on, ja ne pystyvät selittämään kaksoisrakokokeen tuloksen siinä missä kvanttimekaniikka.

John Bellin vuonna 1964 esittämällä epäyhtälöllä on ollut keskeinen rooli siinä, että kvanttimekaniikan outous on hyväksytty ja piilomuuttujateorioiden suosio on jäänyt vähäiseksi. Tämä epäyhtälö on Aspectin, Clauserin ja Zeilingerin työn pohjalla. Olen kirjoittanut Bellin epäyhtälöstä tarkemmin täällä, ja sitä valaistaan myös tiedeakatemian taustamateriaalissa. Askel askeleelta seurattavan selkeän selityksen voi lukea Tanya Bubin ja Jeffrey Bubin sarjakuvasta Totally Random.

Jos luodaan kaksi fotonia, joiden polarisaatio (eli värähtelysuunta) on vastakkainen, niin mittaamalla yhden polarisaation tietää heti toisenkin polarisaation. Tätä ominaisuutta sanotaan lomittumiseksi. (Englanniksi entanglement, kirjaimellisesti yhteenkietoutuminen.) Kvanttimekaniikan mukaan fotonin tila ei ole määrätty ennen kuin sitä mitataan. Piilomuuttujateorioissa fotonien tilat ovat koko ajan määrätyt, emme vain ennen mittaamista tiedä mitkä ne ovat.

Bell hahmotti, että nämä kaksi mahdollisuutta voi erottaa kokeellisesti mittaamalla eri fotonien polarisaatiota eri suunnissa ja tutkimalla tulosten tilastollista riippuvuutta. Missä tahansa teoriassa, jossa fotonien tila on aina määrätty, tämä riippuvuus toteuttaa Bellin epäyhtälön. Kvanttimekaniikassa epäyhtälö rikkoutuu, koska lomittuneen systeemin osat ovat kytköksissä toisiinsa rajattoman pitkien etäisyyksien yli vahvemmin kuin teoriassa, missä systeemin tila on koko ajan määrätty (eikä voida viestiä yli valonnopeudella).

Bellin epäyhtälö on kvanttimekaniikan sääntöjen suoraviivainen seuraus. Noiden yksinkertaisten sääntöjen vieraudesta arkiajattelulle kertoo paljon se, että kesti vuosikymmeniä niiden löytämisestä 1920-luvulla siihen, että Bell esitti nyt nimeään kantavan epäyhtälön.

Teknologista kekseliäisyyttä taasen kuvaa se, että jo vuonna 1972, kahdeksan vuoden kuluttua, Clauser kollegoineen osoitti kokeellisesti, että Bellin epäyhtälö rikkoutuu kvanttimekaniikan ennustamalla tavalla. Tiedeakatemia mainitsee, että yksi ongelma Clauserin tiellä oli se, että hän oli kokeellinen astrofyysikko, joka oli palkattu tekemään radiotähtitiedettä. Clauser sai kuitenkin sovittua, että saa käyttää puolet ajastaan Bellin epäyhtälön testaamiseen. Tämä muistuttaa joustavuuden merkityksestä tutkimusaiheiden muuttamisessa nykyaikoina, missä tutkijoiden oletetaan tietävän tutkimuksensa kulun viisi vuotta etukäteen.

Tiedeakatemia kirjoittaa suurelle yleisölle suunnatussa taustoituksessa, että Clauserin tulosten mukaan ”kvanttimekaniikkaa ei voi korvata millään piilomuuttujateorialla”. Tämä ei ole ihan totta. Tutkijoille suunnattu teksti on huolellisempi, ja siinä selitetään, että tällaisessa piilomuuttujateoriassa tiedon täytyisi kulkea valoa nopeammin (mikä on ristiriidassa suhteellisuusteorian kanssa). Itse asiassa vasta Aspectin ja kollegoiden kehittyneemmät kokeet osoittivat, että tiedon pitäisi kulkea valoa nopeammin, jotta piilomuuttujateoria voisi selittää tulokset.

Bellin epäyhtälöön liittyvä tutkimus on avannut oven kvanttimekaniikan perusteiden hedelmälliselle soveltamiselle teknologiaan. Tiedeakatemia korostaakin perusteluissaan kvanttitietokoneiden ja kvanttikryptografian kasvavaa merkitystä. Näillä aloilla lomittumiseen liittyvien kvanttimekaniikan piirteiden ymmärtäminen ja käyttäminen teknologisesti –missä erityisesti Zeilinger on kunnostautunut– on avainroolissa. Jos Bell ei olisi kuollut vuonna 1990, hänetkin olisi luultavasti palkittu Nobelilla.

Nobelin palkinnot keskittävät paitsi suuren yleisön myös tiedeyhteisön huomiota ja houkuttelevat tutkijoita palkitulle alalle. Gravitaatioaaltojen suora havaitseminen vuonna 2015 ja palkitseminen Nobelilla 2017 ovat tehneet niistä muodikkaan tutkimuskohteen, jota mietitään monissa muissakin yhteyksissä kuin siinä, mikä johti palkintoon. Ehkäpä tämän vuoden palkinto lisää suosiota Bellin epäyhtälön rikkoutumisen ja muiden kvantti-ilmiöiden tutkimiseen myös kosmologiassa.

Kosmisesta inflaatiosta, mikä on ensimmäisenä yhdistänyt kvanttifysiikan ja yleisen suhteellisuusteorian tavalla joka on ennustanut kokeiden tuloksia yksityiskohtaisesti oikein, ei olekaan vielä myönnetty Nobelin palkintoa, vaikka jotkut kosmologit ovat sellaista povanneet.

35 kommenttia “Lomittuneilla fotoneilla”

  1. Erkki Kolehmainen sanoo:

    Suurin molekyyli, jolla kaksoisrakokoe on tehty, sisälsi n. 2000 atomia ja sen molekyylipaino oli n. 25000. Ja tämäkö sitten menee kahden eri raon kautta ja kasautuu uudelleen yhdeksi rakojen jälkeen. Ottaen huomioon, kuinka vaikeaa uusien molekyylien valmistamien saattaa olla, niin ei kuulosta kovin loogiselta! Älkää viitsikö älyttää vanhaa kemstiä!

    1. Syksy Räsänen sanoo:

      Ei, hiukkanen ei hajaudu kahtia ja kasaannu sitten uudelleen. Ei ole olemassa mitään yhdessä paikassa olevaa hiukkasta, vain todennäköisyysjakauma.

      Tämä on todella arkijärjelle vierasta, mikä osoittaa arkiajattelumme vaillinaisuuden.

      1. Erkki Kolehmainen sanoo:

        Arkiajattelu on ollut ainoa luotettava keino selviytyä kaikkien vaarojen ja vuosimiljoonia kestäneíden haasteiden aikana. En ole valmis korvaamaan sitä saduilla tai mielikuvituksella, vaikka niilläkin jotain arvoa on.

        1. Syksy Räsänen sanoo:

          Asia on päinvastoin. Sadut ja arkinen kuvittelu pohjaa arkisen ajattelun käsitteisiin. Fysiikka sen sijaan pohjaa järjestelmällisiin havaintoihin ja matematiikkaan, joka tarjoaa käsitteitä, jotka ovat arkijärjelle vieraita mutta kuvaavat todellisuutta tarkemmin. Se, että todellisuus ei ole arkijärjen mukainen on järkevän epäilyn ulkopuolella.

  2. Markku Kaakkolammi sanoo:

    Tässä puhutaan arktodellisuudesta ja kvanttifysiikasta. Missä on se raja, jonka jälkeen arkielämän havainnot alkavat pitää paikkansa ? Eli että esim. auto on todella tuossa ja että kysymys ei ole todennäköisyysjakaumasta ?

    1. Syksy Räsänen sanoo:

      Tämä onkin tärkeä ja osin ratkaisematon kysymys.

      Kvanttimekaniikan mukaan mitään periaatteellista rajaa ei ole, ainoastaan kvantitatiivinen raja. Arkisen mittakaavan esineiden tilan todennäköisyysjakauma on siis hyvin tiukasti keskittynyt. Ei tosin täysin ymmärretä miten arkinen määrätyn näköinen todellisuus seuraa kvanttimekaniikasta – vai pitääkö kvanttimekaniikkaa muuttaa.

      Tarkemmin, ks.

      https://www.ursa.fi/blogi/kosmokseen-kirjoitettua/kahden-ikkunan-nakoala/

      https://www.ursa.fi/blogi/kosmokseen-kirjoitettua/koopenhaminan-takana/

      http://www.tiede.fi/artikkeli/blogit/maailmankaikkeutta_etsimassa/maarattyina_yhteen

      http://www.tiede.fi/blogit/maailmankaikkeutta_etsimassa/arjen_epatotuus

      1. Jyri T. sanoo:

        Kun kerran itse nostit esiin kysymyksen siitä, miten arkinen määrätyn näköinen todellisuus seuraa kvanttimekaniikasta, niin toivottavasti saanen esittää seuraavan kysymysken/ajatuskokeen:

        Yleinen suhteellisuusteoria on matemaattisesti ”epälineaarinen” kuvaus aika-avaruudesta, kun taas kvanttimekaniikka on täysin ”lineaarinen” kuvaus aaltofunktioiden maailmasta.

        Mitä jos makroskooppinen (”lokalisoitunut”) todellisuus nousee/syntyy siitä, että aaltofunktion on pakko romahtaa, kun se kohtaa riittävästi aika-avaruuden ”epälineaarisuuksia”?

        Esimerkiksi makroskooppisen hiukkasen oma gravitaatio riittäisi aiheuttamaan niin paljon epäjatkuvuutta ajan kulkuun, että sen aaltofunktio ei enää voisi pysyä koherenttina vaan olisi pysyvästi ”romahtanut” ja hiukkanen pysyisi ”lokalisoituneena” eli ”makroskooppisessa tilassa” koko ajan.

        Toisaalta, mikä aiheuttaisi tällaisen epäjatkuvuuden esim. silloin, kun alkeishiukkanen ”valitsee” osumakohdan kaksoisrakokokeessa?

        Onko tällaista ajatusta pohdittu?

        1. Syksy Räsänen sanoo:

          Onpa hyvinkin. Roger Penrose (joka sai vuonna 2020 Nobelin palkinnon sen osoittamisesta, että mustat aukot ovat yleisen suhteellisuusteorian yleinen ennustus) on tunnetuin tällaisia ideoita esittänyt tutkija. Tätä on kokeellisestikin tutkittu:

          https://www.quantamagazine.org/physics-experiments-spell-doom-for-quantum-collapse-theory-20221020/

  3. Markku Tamminen sanoo:

    Erkki Kolehmainen on mielestäni puolittain oikeassa.

    Voi olla, että kvanttifysikaalisesti kuvatun kissan ja naapurin katin välillä on jokin toistaiseksi ylittämätön käsitteellinen kuilu, mutta en ymmärrä, mitä tekemistä tällä on todellisuuden tai harhan kanssa. Eiköhän kysymys ole havaitsemisen tasoista ja kuvausten tarkoituksenmukaisuudesta. On itse asiassa melko absurdi ajatus, että arkielämämme olisi jotenkin perustavalla tavalla harhaista. Syksyn muuten ansiokkaissa esityksissä tämä ajatus on aina tuntunut minusta todella oudolta, enkä ole löytänyt sille kunnon perusteluja. Ainoa johtopäätös lienee, että kysymyksessä on äärimmilleen viety fysikalismi, siis filosofinen kannanotto, joka ei perustu fysiikkaan.

    1. Syksy Räsänen sanoo:

      Arkiajatteluumme kuuluu esimerkiksi ajatus, että asioiden tila on määrätty. Kvanttimekaniikan mukaan tämä ei pidä paikkaansa.

      Myös suhteellisuusteoriasta löytyy (vielä vankemmalla pohjalla olevia) esimerkkejä siitä, että arkikäsityksemme maailmasta on perusteiltaan virheellinen, ks. esim. https://www.ursa.fi/blogi/kosmokseen-kirjoitettua/maljat-ennen-vai-jalkeen/

    2. Erkki Kolehmainen sanoo:

      Juuri näin. Fyysikko ei tarvitse mitään ihmeaivoja. Kun Einstein kuoli, niin patologit ryntäsivät tutkimaan hänen aivojaan, mutta ei sieltä mitään ihmeellistä löytynyt – samaa harmaata ja valkoista massaa kuin muillakin. Ei edes niiden koko ollut tavallista suurempi. Syksyn asenne on tyypillistä ihmiselle, joka haluaa nähdä työnsä erityisen arvokkaana ja merkittävänä. Toki sellainen ajattelu on luvallista (ja tavallista) ellei se johda muiden vähättelyyn!

  4. On hyvä että kvantti-ilmiöt saavat julkisuutta, koska niissä on vielä paljon oppimista ja sulateltavaa. Vaikka kvanttimekaniikan tiedetään olevan totta, maailmankuvallisesti ollaan vähän juututtu jonnekin 1800-luvun alun kellokoneistouniversumiin. Vaikka asian varsinainen ymmärtäminen onkin vaikeaa, se on jo hyvä askel eteenpäin että tulee ihmetelleeksi miten maailmankaikkeuden aaltofunktiosta syntyy havaintokokemus klassista fysiikkaa noudattavasta kehosta joka on osa ympäröivää kaikkeutta. Ehkä kvanttifysiikka itsessään on selvä juttu, mutta maailma ei!

  5. Cargo sanoo:

    Jos oletetaan,että aika ja avaruus ovat jotenkin seurausta energiasta ja vuorovaikutuksesta, niin eipä tuo lomittuminen ole psykologisesti mikään mahdoton asia: jos hiukkaset ovat osa samaa kvanttimekaanista systeemiä, niin muutokset voivat tapahtua riippumatta ulkoisesta ajasta ja avaruudesta.

    Mutta voisiko Dr. Räsänen valaista meitä uteliaita maallikoita siitä, että mitä yksöis- tai kaksoisrakokokeessa oikein tapahtuu? Kun siis hiukkasta kuvaava aalto tulee raolle, mikä aiheuttaa tilan muutoksen, niin kuka/ketkä sitä hiukkasta oikein havaitsee ja minne se tieto tallentuu? Ja kun hiukkanen jatkaa aaltomaisesti matkaansa ja lopulta osuu havaintolevylle, niin mikä sen paikallistumisen aiheuttaa ja vastaako se jotenkin sitä, että hiukkasen aalto sujahtaa äärimmäisen kapeaan ’yksöisrakoon’, mikä nostattaa suuren, välähdyksenä ilmenevän liikemäärän?

    Lisäksi, tarkoittaako tuo kvanttimekaaninen aalto ylipäätään yhtään mitään, jos se ei vuorovaikuta ympäristönsä kanssa? Tuli meinaan Descartesin ”ajattelen, siis olen” lausahduksesta mieleen modernimpi versio: ”vuorovaikutan, siis olen” 🙂

    1. Syksy Räsänen sanoo:

      Lomittuminen ei liity ajan ja avaruuden luonteeseen.

      Kaksoisrakokokeesta, ks. https://www.ursa.fi/blogi/kosmokseen-kirjoitettua/kahden-ikkunan-nakoala/

    2. Jos olemassaololla viitataan mitattavaan olemassaoloon eli havaintoon, niin havainto vaatii vuorovaikutuksen, joten siinä mielessä ”vuorovaikutan, siis olen” voisi olla tämän järkevältä kuulostava yleistys. Kuitenkin muistaen että vuorovaikutuskaan ei ole absoluuttinen käsite, esimerkiksi koska on tilanteita jossa samaa ilmiötä voidaan kuvata kahdella yhtäpitävällä (kvantti)teorialla, joista toinen on vahvasti ja toinen heikosti vuorovaikuttava.

      Kvanttiteoria selittää havainnot, mutta ei havaitsijaa. Havaitsijaa on vaikea mallintaa, koska se vaikuttaisi olevan tolkuttoman monimutkainen kvanttitila. Tai ehkä se on koko maailmankaikkeus.

    3. Eusa sanoo:

      Vuorovaikutus on ontologisesti fysikaalisuuden keskiössä. Lomittumiskorrelaatiokin voidaan todentaa vasta kun tehdään vertailu vaihtamalla tietoa vuorovaikutussignaalein.

      Mittaamisen fudamentaali haaste puolestaan on se, että mittalaite on osa mitattavaa vuorovaikutusjatkumoa.

  6. miguel sanoo:

    Minusta on outoa, että kvanttifysiikan ilmiöiden todennäköisyysjakauma tiivistyy, kun tullaan arkielämän ilmiöihin. Äkkiä luulisi, että kun muuttujien määrä kasvaa, niin todennäköisyysjakauma leviää entisestä. Ihan kuin kvantti-ilmiöillä olisi ”pyrkimys determinismiin” isommassa skaalassa ja koko kvanttifysiikka olisi jotenkin skaalautuva. Sille ei varmaan ole mitään teoriaa tai perustetta?

    Sitten kysyisin vielä, että on ihan selvä, että jos fotonipari syntyy ja toinen on sininen ja toinen on punainen, niin kun tiedämme toisen värin, niin tiedämme toisen, mutta voidaanko tuo spin siis muuttaa, eli voidaan maalata punainen pallo siniseksi, jolloin toisen fotonin spin muuttuu samalla hetkellä. Jotenkin arkijärkeen sopisi joku oskillointi, jossa spinit sykronoituvat syntyhetkellä ja muutoksessa emme oikeastaan muuta mitään, vain havaitsemme tietyn spinin, joka olisi vaihtunut ilman mitään väliintuloa, ja mitään informaatiota ei liiku.

    Ymmärrän toki, että kvanttifysiikan puolesta puhuu moni muukin asia. Tämä ei ole mikään oma nojatuoliteoria, vaan kysymys.

    1. Martti V sanoo:

      Kvanttifysiikan ilmiöt ovat tilastollisia. Hiukkasten tila on epämääräinen ja makroskooppisesti havaitsemme keskiarvotilan. Kyse ei ole determinisyydestä. Pohjimmiltaan kaikki perustuu kvanttikenttiin ja hiukkaset ovat niiden satunnaisia eksitaatioita.

  7. Erkki Kolehmainen sanoo:

    ”,,,koska lomittuneen systeemin osat ovat kytköksissä toisiinsa rajattoman pitkien etäisyyksien yli vahvemmin kuin teoriassa, missä systeemin tila on koko ajan määrätty (eikä voida viestiä yli valonnopeudella).”

    Jos ”lomittumisesta” todella seuraa mahdollisuus viestiä yli valonnopeudella, niin se ei voi pitää paikkaansa eikä sellaista voi olla olemassa. Syksyn ansioksi on sanottava, että hän kirjoittaa tekstiinsä argumentin, joka tekee tyhjäksi v. 2022 fysiikan Nobel-palkinnon perusteet ja hänen aiemmat pohdiskelunsa lomittumisesta!

    1. Syksy Räsänen sanoo:

      Lomittumiseen ei liity viestimistä yli valonnopeudella.

      1. Martti V sanoo:

        Voidaanko tätä todistaa? Kahden havainnoitsijan eri paikassa pitäisi saman aikaisesti todeta tila. Toisaalta toinen havaitsija pystyy kertomaan tilasta valonnopeudella, jonka aikana myös hiukkaset pystyvät vuorovaikuttamaan. Toinen vaihtoehto on, että toisen lomittuneen hiukkasen tilan mittaus määrää myös toisen tilan. Lähtökohtaisesti tilat ovat satunnaisia. Ehkei tila ole pohjimmiltaan satunnainen vaan ennalta määrätty jotenkin, mitä ei ymmärretä.

        1. Syksy Räsänen sanoo:

          Lomittumiseen ei kvanttimekaniikassa liity viestimistä yli valonnopeudella. Kvanttimekaniikan mukainen lomittuminen selittää havainnot, ja avainasemassa on se, että systeemin tila ei ole aina määrätty.

          Jos havainnot haluaisi selittää teorialla, jossa systeemin tila on määrätty, se vaatisi sitä, että pitää olla mahdollista välittää tietoa yli valonnopeudella. (Toinen vaihtoehto on se, että kaikki maailmankaikkeuden tapahtumat kaikkialla ajassa ja avaruudessa on niin vahvasti määrätty, että mitään paikallisia luonnonlakeja ei ole.)

          Tämän vuoden Nobelin palkinnon saajat todistivat tämän kokeellisesti.

          1. Cargo sanoo:

            Miten herrat Aspect, Clauser ja Zeilinger varmistivat sen, etteivät heidän havaintokojeensa olleet jollakin salakavalalla tavalla kytköksissä toisiinsa ja siten sotkeneet mittauksia (esim. kummankin mittalaitteen käyttämät satunnaisluvut korreloisivat jotenkin)? Miten ylipäätään voidaan sulkea pois muuttujia, jos niiden luonteesta ei ole mitään havaittavaa tietoa. Einstein olisi varmasti keksinyt jonkin ajatuskokeen, joka olisi saanut nobelistitriolle jauhot suuhun 😀

            Itse vähän arvailen, että Einsteinin epäluuloisuuden taustalla oli hänen rakas suhde termodynamiikkaan, jonka avulla hän sai oman nobelinsa. Kvanttimekaniikkahan on päällisin puolin termodynaaminen teoria, joka antaa keskimäärin oikeita tuloksia, mutta kuten klassisessa termodynamiikassa, pinnan alta tulisi löytyä liikeratojen kaltaista realismia.

          2. Syksy Räsänen sanoo:

            Tällaisten mahdollisuuksien poissulkeminen on ollut merkittävä osa työtä.

            Kokeen yhdessä versiossa kokeen asetukset määritellään eri päissä käyttäen fotoneita, jotka ovat tulleet miljardien valovuosien takaa, ja niin nopeasti, että valo ei ehdi kulkea päiden välillä. Kytköksen pitäisi siis ulottua miljardien valovuosien päähän paikassa ja miljardien vuosien päähän ajassa. Tällaista mahdollisuutta ei voine periaatteessakaan sulkea pois, mutta tuntuu erittäin vaikealta rakentaa teoriaa, jossa kaikki maailmankaikkeuden tapahtumat ovat tällä tavalla yhteydessä, ja joka olisi silti sopusoinnussa havaintojen kanssa.

          3. Einsteinin ”Jumala ei heitä noppaa” kuulostaa kritiikiltä Kööpenhaminan tulkintaa kohtaan, joka siinä vaiheessa olikin ainoa tulkinta. Monimaailmatulkinta tuli vasta pari vuotta Einsteinin kuoleman jälkeen. Ehkä hän olisi tykännyt siitä enemmän, kun siinä ei esiinny satunnaislukuja.

  8. Lomittumiseen ei liity kommunikaatiota yli valonnopeudella. Kuitenkin sitä käyttämällä joissakin peleissä voidaan pelata niin vahvasti, että kvanttimekaniikasta tietämätön tarkkailija luulee että pelaajat huijaavat kommunikoimalla keskenään. Eli ulkoisen tarkkailijan mielestä yliluonnollista kommunikaatiota tapahtuu, vaikka toimijoiden itsensä näkökulmasta ei. Näille ajatuskokeille on annettu nimi pseudotelepatia. Niiden viimeaikaisista käänteistä olisi mukava kuulla joskus lisää. Lähde: wikipedia:Quantum_pseudo-telepathy. Niitä pelejä oli tuolla sivulla aiemmin vain yksi, mutta nyt siellä on toinenkin (GHZ).

  9. Kas sanoo:

    Katoaako fotonin polarisaatio fotonin joutuessa mustaan aukkoon, ts onko mustalla aukolla jonkinlainen fotonin aiheuttama ”polarisaatio-informaatio” tai onko musta aukko täysi polarisaatiosta vapaa?

    Kysymys liittyy ajatusleikkiin, jossa kaksi fotonia on lomittunut ja toinen fotoni joutuu mustaan aukkoon.

    Jos fotonin mustaan aukkoon joutumisen jälkeen mitataan mustan aukon ulkopuolella olevan fotonin polarisaatio, niin tiedetäänkö myös tässä tilanteessa mustaan aukion joutuneen polarisaatio (ja pitäisikö sen jotenkin ”näkyä” mustan aukon toiminnassa)?

    Ja jos oletetaan mustan aukon ajan myötä haihtuvan Hawkingin säteilyn myötä, niin tuleeko polarisaatio-informaatio sen myötä ”ulos” mustasta aukosta?

    1. Syksy Räsänen sanoo:

      Mustalla aukolla on pyörimismäärä, luulisin että polarisaatio vaikuttaa siihen. Ei tiedetä, mitä tapahtuu mustaan aukkoon joutuneelle informaatiolle, etenkin kun lomittuminen otetaan huomioon, koska ei täysin osata yhdistää mustia aukkoja kuvaavaa yleistä suhteellisuusteoriaa ja lomittumista kuvaavaa kvanttifysiikkaa. Asiaa on tutkittu paljon, mutta varmuutta ei ole.

  10. Helena Othman sanoo:

    Vääntäisitkö rautalangasta, jos informaatio ei (edelleenkään) siirry valoa nopeammin, ”mikä” jos mikään siirtyy kun toista hiukkasta tarkkaillaan ja sen pari myöskin ilmaisee positionsa (tarkkailusta johtuen)?
    Onko lomittuminen pikemminkin jakaantuneen tai jakaantuneiden hiukkasten ominaisuus kuin ”informaationsiirto”?

    1. Syksy Räsänen sanoo:

      ”Onko lomittuminen pikemminkin jakaantuneen tai jakaantuneiden hiukkasten ominaisuus kuin ”informaationsiirto”?” Asia on juuri näin. Lomittunut hiukkaspari muodostaa erottamattoman kokonaisuuden. Kun yhden tila määräytyy, myös toisen tila määräytyy (eli yksi sen mahdollisista vaihtoehdoista valikoituu). Koska on sattumanvaraista, mikä vaihtoehdoista valikoituu, tämä määräytyminen ei välitä informaatiota.

      1. Lentotaidoton sanoo:

        Mielestäni monilla ihmisillä on (jokin pakonomainen) käsitys että lomittumisessa olisi kyse kahdesta eri systeemistä (jotka olisivat ”kietoutuneet”). Lomittunut tilahan on yksi kokonaisuus, mikä sitten dekoherenssissä randomisti romahtaa yhdellä kertaa. Silloin on selvää ettei informaationsiirtoa hiukkaselta toiselle tarvita, ei lokaalia eikä ei-lokaalia (vaikka etäisyys hiukkasten välillä olisi suurikin).

        1. Eusa sanoo:

          Toki ihmistä kiinnostaisi tietää tarkemmin mikä syvällinen mekanismi säilyttää korrelaation. Onko se jokin avaruusajan rytmi, joka säilyttää kvanttitilojen vastakkaisuuden vai mikä? Silmukkakvanttigravitaation haastava kysymys on kuinka edes liikemäärä säilyy. Fundamentaalissa fysiikassa on vielä loppumattomasti tekemistä.

          1. Syksy Räsänen sanoo:

            Kvanttimekaniikassa lomittumisella ei ole aika-avaruuden rakenteen kanssa mitään tekemistä.

            Siitä, mikä on perustavanlaatuisempi teoria kvanttifysiikan takana ei tiedetä.

            Liikemäärän säilyminen ei liity tähän, se riippuu siitä millainen aika-avaruus on, se tunnetaan yleisen suhteellisuusteorian puolelta hyvin. Yleistä suhteellisuusteoriaa ei tosin osata kokonaisuudessaan yhdistää kvanttiteoriaan, silmukkakvanttigravitaatio on yksi yritys tehdä niin.

Vastaa

Sähköpostiosoitettasi ei julkaista. Pakolliset kentät on merkitty *